Managerial Accounting in The Information Age: True-False

You might also like

Download as pdf or txt
Download as pdf or txt
You are on page 1of 106

CH APTE R 1

Managerial Accounting in the Information Age


TRUE-FALSE

1. Managerial accounting stresses accounting concepts and procedures that are relevant to
preparing reports for internal users of accounting information.

2. The goal of managerial accounting is to provide information for planning, controlling and
decision making.

3. A thorough understanding of managerial accounting is essential for an effective manager.

4. A budget informs managers of planned production amounts and the cost of resources
needed for production.

5. Budgets are financial plans prepared by managerial accountants.

6. Only amounts that can be expressed in dollars and cents can be used in preparing
budgets.

7. An unfavorable evaluation of an operation indicates that the manager of that operation is


not performing adequately.

8. Performance reports are used for control purposes.

9. Performance reports, like other managerial accounting reports, must follow GAAP.

10. Performance reports may show comparisons of current period performance to the
planned, or budgeted, performance.

11. Managers need to investigate every difference between actual and budgeted costs in a
performance report.

12. Decisions to reward or punish managers are not part of the planning and control process.

13. Managerial accounting is directed at external users of accounting information.

14. Managerial accounting must follow generally accepted accounting principles.

15 Managerial accounting may present more detailed information than financial accounting.

16. Managerial accounting is optional and stresses that the information provided should be
useful to managers.

17. Financial accounting is concerned with presenting results of past transactions while
managerial accounting places considerable emphasis on the future.
18. Variable costs increase or decrease in total in proportion with changes in the level of
business activity.

19. Insurance is generally a controllable cost for a factory department supervisor.

20. Variable cost per unit remains the same even though there is a change in the number of
units produced.

21. Fixed cost per unit remains constant when the number of units produced changes.

22. Sunk costs are a significant consideration in incremental analysis.

23. Opportunity costs are the value of benefits foregone when one alternative is selected over
another.

24. Direct costs are directly traceable to a product, activity, or department.

25. A manager can influence a controllable cost.

26. Incremental analysis involves calculating the difference in revenue and difference in
costs between alternatives.

27. The actions of a manager are influenced by the performance measures that are used to
evaluate the manager.

28. In general, having a single performance measure by which managers are evaluated will
lead to financial success for a company.

29. A good single measure of performance for a sales force would be the ratio of sales to new
customers to total sales.

30. Costs that do not increase or decrease due to a special order are never considered
incremental costs for the special order decision.

31. The current business era is referred to as the information age.

32. Advances in technology make it easier for potential buyers to compare prices globally.

33. The value chain includes the company and its suppliers and customers.

34. Businesses sometimes share sales databases with suppliers so suppliers can respond more
quickly.

35. Enterprise resource planning systems focus on automating customer service and support.
Chapter 1 Managerial Accounting in the Information Age 1-3

36. Enterprise resource planning systems (ERP) often support accounting, human resources,
and e-commerce, in addition to production.

37. Supply chain management systems (SCM) allow suppliers some access to a company’s
databases so goods can more profitably be delivered to a company’s customers.

38. Customer Relationship Management Systems (CRM) automate customer service and
support.

39. A Customer Relationship Management System (CRM) might allow a customer to track
his/her package as it is being shipped across the country.

40. Dell Computer’s web site that lets you keep track of your order being built on a daily
basis is an example of ERP (Enterprise Resource Planning).

41. All ethical dilemmas have a single correct solution.

42. When making ethical choices, one question you should ask yourself is: “Which
alternative will do the most good or the least harm?”

43. The Institute of Management Accountants is primarily responsible for determining


GAAP.

44. The Sarbanes-Oxley Act requires that companies provide relevant managerial accounting
information to decision-makers.

45. In most organizations, the treasurer is the top managerial accountant.

46. The treasurer usually reports to the controller.

47. The treasurer is responsible for preparing reports for planning and evaluating company
activities.

48. The controller has custody of cash and funds invested in marketable securities.

Answers
1 T 9 F 17 T 25 T 33 T 41 F
2 T 10 T 18 T 26 T 34 T 42 T
3 T 11 F 19 F 27 T 35 F 43 F
4 T 12 F 20 T 28 F 36 T 44 F
5 T 13 F 21 F 29 F 37 T 45 F
6 F 14 F 22 F 30 T 38 T 46 F
7 F 15 T 23 T 31 T 39 T 47 F
8 T 16 T 24 T 32 T 40 F 48 F
MULTIPLE CHOICE

49. Managerial accounting stresses accounting concepts and procedures that are relevant to
preparing reports for
A. taxing authorities.
B. internal users of accounting information.
C. external users of accounting information.
D. the Securities and Exchange Commission (SEC).

50. The goal of managerial accounting is to provide information that managers need for
A. planning.
B. control.
C. decision making.
D. All of the above answers are correct.

51. The financial plans prepared by managerial accountants are referred to as


A. budgets.
B. financial statements.
C. treasurer’s reports.
D. controller’s opinions.

52. Which of the following is not a reason that actual results may differ from the company’s
plan?
A. The plan may not have been followed properly.
B. The plan may not have been well thought-out.
C. Changing circumstances may have made the plan out of date.
D. All of the above are reasons that actual results may differ from the company’s
plan.

53. It is possible for a manager to receive a positive evaluation when the operation receives
a(n)
A. favorable evaluation.
B. neutral or mixed evaluation.
C. unfavorable evaluation.
D. All of the above answers are correct.

54. The last step in the planning and control process is to


A. implement the plan.
B. construct the plan.
C. make decisions based on the evaluation of the results.
D. compare actual results to the planned results.
Chapter 1 Managerial Accounting in the Information Age 1-5

55. Performance reports often compare current period performance with


A. performance in a prior period.
B. planned (budgeted) performance.
C. Both A and B are correct.
D. Neither A nor B is correct.

56. A difference between actual costs and planned costs


A. should be investigated if the amount is exceptional.
B. indicates that the planned cost was poorly estimated.
C. indicates that the manager is doing a poor job.
D. should be ignored unless it involves the cost of ingredients.

57. The principle that managers follow when they only investigate departures from the plan
that appear to be significant is commonly known as
A. small amounts don’t matter.
B. management by exception.
C. only labor and materials deserve attention.
D. exceptional costs yield exceptional results.

58. Below is a performance report that compares budgeted and actual profit of Mandarin
Smoothie for the month of June:

Budget Actual Difference


Sales $180,000 $182,000 $2,000
Less:
Cost of ingredients 142,000 146,000 (4,000)
Salaries 11,000 11,200 (200)
Controllable profit $27,000 $24,800 ($2,200)

In evaluating the department in terms of its increases in sales and expenses, what will be
most important to investigate?
A. Sales
B. Cost of ingredients
C. Salaries
D. All three components have equal importance.

59. Managerial accounting


A. is primarily directed at external users of accounting information.
B. is required by taxing authorities such as the IRS.
C. must follow GAAP.
D. is optional.
60. The fundamental difference between managerial and financial accounting is that
A. all financial accounting information is audited by Certified Public Accountants
whereas managerial accounting information is not audited by anyone.
B. managerial accounting is concerned principally with determining the cost of
inventory (ending inventory and cost of goods sold), whereas financial accounting
is concerned with a wider range of the organization’s activities.
C. managerial accounting provides information for decision-makers within the
organization, whereas financial accounting provides information for individuals
and institutions external to the organization.
D. financial accounting information follows U.S. Generally Accepted Accounting
Principles, whereas managerial accounting information generally follows rules set
forth by the Institute of Management Accountants.

61. Which of the following is not a difference between financial accounting and managerial
accounting?
A. Financial accounting is primarily concerned with reporting the past, while
managerial accounting is more concerned with the future.
B. Managerial accounting uses more nonmonetary information than is used in
financial accounting.
C. Managerial accounting is primarily concerned with providing information for
external users while financial accounting is concerned with internal users.
D. Financial accounting must follow GAAP while managerial accounting is not
required to follow GAAP.

62. Which of the following is most likely to make use of Spruce Company’s managerial
accounting information?
A. the IRS
B. an individual contemplating an investment in Spruce Company
C. a company that is one of Spruce’s main competitors
D. the production manager of Spruce’s plant in Minnesota

63. Which of the following costs does not change when the level of business activity
changes?
A. total fixed costs
B. total variable costs
C. total direct materials costs
D. fixed costs per unit

64. Variable cost per unit


A. increases when the number of units produced increases.
B. does not change when the number of units produced increases.
C. decreases when the number of units produced increases.
D. decreases when the number of units produced decreases.
Chapter 1 Managerial Accounting in the Information Age 1-7

65. Uno Pizza produced and sold 800 pizzas last month and had total variable ingredients that
cost $3,440. If production and sales are expected to increase by 10% next month, which
of the following statements is true?
A. Total variable materials costs are expected to be $3,784
B. Variable material cost per unit is expected to be $4.73
C. Total variable materials costs are expected to be $3,444.30
D. Total variable materials costs are expected to be $344

66. A company has a cost that is $2.00 per unit at a volume of 12,000 units and $2.00 per unit
at a volume of 16,000 units. What type of cost is this?
A. fixed
B. variable
C. sunk
D. incremental

67. Which of the following is not likely to be a fixed cost?


A. direct materials
B. rent
C. depreciation
D. salary of the human resources director

68. Marco Diner produced and sold 2,000 bagels last month and had fixed costs of $6,000. If
production and sales are expected to increase by 10% next month, which of the following
statements is true?
A. Total fixed costs will increase.
B. Total fixed costs will decrease.
C. Fixed cost per unit will increase.
D. Fixed cost per unit will decrease.

69. Which of the following statements regarding fixed costs is true?


A. When production increases, fixed cost per unit increases.
B. When production decreases, total fixed costs decrease.
C. When production increases, fixed cost per unit decreases.
D. When production decreases, total fixed costs increase.

70. Costs incurred in the past which are not relevant to present decisions are
A. fixed costs.
B. sunk costs.
C. opportunity costs.
D. indirect costs.
71. A sunk cost is a cost
A. incurred in the past which is not relevant to present decisions.
B. incurred in the current period which changes with changes in production activity.
C. incurred in the current period which remains constant even though production
activity changes.
D. which is estimated to occur in the future.

72. Sunk costs


A. are not relevant for decision making
B. would include the cost of your tuition after the refund deadline has passed.
C. are costs that have been incurred in the past.
D. All of the above are correct.

73. Opportunity costs are


A. considered to be fixed costs in the short term.
B. another term for sunk costs.
C. able to be controlled by most effective managers.
D. the value of benefits foregone when one decision is selected over another.

74. The benefits that are given up when another alternative is selected is a(n)
A. sunk cost.
B. controllable cost.
C. opportunity cost.
D. direct cost.

75. You own a car and are trying to decide whether or not to trade it in and buy a new car.
Which of the following costs is an opportunity cost in this situation?
A. the trip to Cancun that you will not be able to take if you buy the car
B. the cost of the car you are trading in
C. the cost of your books for this term
D. the cost of your car insurance last year

76. A retailer purchased some trendy clothes that have gone out of style and must be marked
down to 40% of the original selling price in order to be sold. Which of the following is a
sunk cost in this situation?
A. the current selling price
B. the original selling price
C. the original purchase price
D. the anticipated profit

77. A cost which is directly traceable to a product, activity, or department is a(n)


A. fixed cost.
B. managerial cost.
C. opportunity cost.
D. direct cost.
Chapter 1 Managerial Accounting in the Information Age 1-9

78. Which of the following statements regarding direct and indirect costs is true?
A. The amount of direct costs in a department is always less than the amount of
indirect costs in that department.
B. A department with no variable costs will also have no direct costs.
C. The distinction between a direct and indirect cost depends on the object of the
cost tracing.
D. If a cost is indirect to a department within a plant, it will also be indirect for the
plant as a whole.

79. Which of the following is a direct cost in relation to the cost of teaching the managerial
accounting course you are currently taking?
A. The cost of the paper that you receive as handouts for the class
B. The cost of the room you are using for the class
C. The cost of the registration system that allowed you to enroll in the class
D. The cost of the financial aid department that helps you fund the cost of taking the
class

80. Which of the following is likely to be a noncontrollable cost of a department supervisor?


A. labor in the department
B. materials used in the department
C. insurance on the plant
D. overtime premium pay earned by those working in the department

81. A manager should be evaluated based on


A. noncontrollable costs.
B. opportunity costs.
C. controllable costs.
D. sunk costs.

82. Shula’s 347 Grill has budgeted the following costs for a month in which 1,600 steak
dinners will be produced and sold: Materials, $4,080; hourly labor (variable), $5,200; rent
(fixed), $1,700; depreciation, $800; and other fixed costs, $600. Each steak dinner sells
for $14.00 each. How much is the budgeted variable cost per unit?
A. $5.80
B. $7.74
C. $6.68
D. $3.25

83. Shula’s 347 Grill has budgeted the following costs for a month in which 1,600 steak
dinners will be produced and sold: Materials, $4,080; hourly labor (variable), $5,200; rent
(fixed), $1,700; depreciation, $800; and other fixed costs, $600. Each steak dinner sells
for $14.00 each. What is the budgeted total variable cost?
A. $5,200
B. $9,280
C. $10,080
D. $2,300
84. Shula’s 347 Grill has budgeted the following costs for a month in which 1,600 steak
dinners will be produced and sold: Materials, $4,080; hourly labor (variable), $5,200; rent
(fixed), $1,700; depreciation, $800; and other fixed costs, $600. Each steak dinner sells
for $14.00 each. What is the budgeted total fixed cost?
A. $7,180
B. $1,700
C. $2,300
D. $3,100

85. Shula’s 347 Grill has budgeted the following costs for a month in which 1,600 steak
dinners will be produced and sold: Materials, $4,080; hourly labor (variable), $5,200; rent
(fixed), $1,700; depreciation, $800; and other fixed costs, $600. Each steak dinner sells
for $14.00 each. What is the budgeted fixed cost per unit?
A. $1.06
B. $1.44
C. $4.49
D. $1.94

86. Shula’s 347 Grill has budgeted the following costs for a month in which 1,600 steak
dinners will be produced and sold: Materials, $4,080; hourly labor (variable), $5,200; rent
(fixed), $1,700; depreciation, $800; and other fixed costs, $600. Each steak dinner sells
for $14.00 each. What is Shula’s budgeted profit?
A. $22,400
B. $13,120
C. $10,020
D. $12,380

87. Shula’s 347 Grill has budgeted the following costs for a month in which 1,600 steak
dinners will be produced and sold: Materials, $4,080; hourly labor (variable), $5,200; rent
(fixed), $1,700; depreciation, $800; and other fixed costs, $600. Each steak dinner sells
for $14.00 each. How much would Shula’s profit increase if 10 more dinners were sold?
A. $140.00
B. $62.60
C. $58.00
D. $82.00

88. Ceradyne projects variable labor costs of $21,500 in July when 8,600 units are produced.
If production is expected to drop to 8,000 units in August, what is the expected labor cost
in August?
A. $21,500
B. $20,000
C. $23,113
D. $20,900
Chapter 1 Managerial Accounting in the Information Age 1-11

Answers
49 B 65 A 81 C
50 D 66 B 82 A
51 A 67 A 83 B
52 D 68 D 84 D
53 D 69 C 85 D
54 C 70 B 86 C
55 C 71 A 87 D
56 A 72 D 88 B
57 B 73 D
58 B 74 C
59 D 75 A
60 C 76 C
61 C 77 D
62 D 78 C
63 A 79 A
64 B 80 C
CH APTE R 4
Cost-Volume-Profit Analysis
TRUE-FALSE

1. Total variable costs change in direct response to changes in volume or activity.

2. Fixed costs per unit remain the same when the level of activity changes.

3. Total fixed costs remain the same when the level of activity changes within the relevant
range.

4. Total fixed costs remain constant across all levels of activity.

5. Mixed costs are also referred to as semivariable costs.

6. A step cost is similar to a fixed cost, except that the relevant range is smaller for a step
cost than a fixed cost.

7. Direct materials and direct labor are examples of fixed costs.

8. Variable costs are the same amount per unit at any activity level.

9. Fixed costs are the same amount per unit at any activity level.

10. Discretionary fixed costs cannot be easily changed in a relatively brief period of time.

11. Total production cost is generally a mixed cost.

12. In order to use C-V-P analysis, costs must be broken out into fixed and variable
components.

13. The account analysis method of estimating fixed and variable costs uses a computer
program to fit a line to historical data.

14. The account analysis method is subjective in that different managers viewing the same
set of facts may reach different conclusions regarding which costs are fixed and which
costs are variable.

15. The high-low method of estimating fixed and variable cost components generally
provides more accurate results than regression analysis.
4-2 Test Bank to accompany Jiambalvo Managerial Accounting, 4th Edition

16. The high-low method fits a straight line to the data points that represent the highest and
lowest levels of activity.

17. When using the high-low method, the fixed cost is found by calculating intercept point of
the sloped line on the vertical axis.

18. Total cost equals fixed cost plus variable cost per unit times the activity level in units.

19. Computer software is usually used to conduct a regression analysis.

20. Total costs and activity are assumed to have a linear relationship within the relevant
range.

21. Account analysis is not generally used to estimate fixed and variable costs.

22. A significant weakness of the high-low method is that it uses only two data points.

23. A significant weakness of regression analysis is that it uses only two data points.

24. In the real world, all cost behavior patterns are linear.

25. Profit is equal to revenue plus variable costs plus total fixed costs.

26. At the break-even point, total revenue equals total fixed costs.

27. The margin of safety is the difference between the current level of sales and break-even
sales.

28. The contribution margin is fixed costs minus total variable costs.

29. If the contribution margin ratio is 40%, it means that every $1.00 of sales will contribute
$0.40 to covering fixed costs and generating a profit.

30. Contribution margin ratio is generally the same as gross margin ratio.

31. At the breakeven point, total fixed expenses equal total contribution margin.

32. The margin of safety is the difference between the expected level of sales and break-even
sales.

33. If X = the number of units sold, profit = (contribution margin per unit times X) less total
fixed costs.

34. Contribution margin ratio = (selling price less fixed expenses) divided by selling price.

35. When performing cost-volume-profit analysis with multiple products, it is important to


assume the sales mix remains constant.

36. CVP analysis can only be performed for a firm which sells a single product.
Chapter 4 Cost-Volume-Profit Analysis 4-3

37. Total fixed costs divided by the contribution margin ratio equals the breakeven point.

38. When performing multiple product CVP analysis, a necessary assumption is that the sales
mix remains constant.

39. The higher the operating leverage is, the more responsive profit will be to changes in
sales volume.

40. Firms that have relatively high levels of fixed costs have low operating leverage.

41. Firms have no control over their level of operating leverage.

42. Firms that have high operating leverage tend to avoid large fluctuations in profit when
sales fluctuate.

43. When dealing with constrained resource situation, a company should generally produce
only the product with the highest contribution margin per unit.

44. When dealing with constrained resource situation, a company should generally produce
only the product with the highest contribution margin ratio.

45. When there is a constraint on how many units can be produced, the focus shifts from
contribution margin per unit to contribution margin per unit of the constraint.

46. When there is a constraint on how many units can be produced, the product with the
highest contribution margin per unit should be produced.

47. The R Square is a statistical measure of how well the regression line fits the data.

Answers

1 T 13 F 25 F 37 T
2 F 14 T 26 F 38 T
3 T 15 F 27 T 39 T
4 F 16 T 28 F 40 F
5 T 17 F 29 T 41 F
6 T 18 T 30 F 42 F
7 F 19 T 31 T 43 F
8 T 20 T 32 T 44 F
9 F 21 F 33 T 45 T
10 F 22 T 34 F 46 F
11 T 23 F 35 T 47 T
12 T 24 T 36 F
4-4 Test Bank to accompany Jiambalvo Managerial Accounting, 4th Edition

MULTIPLE CHOICE

48. Which of the following is most likely to be a variable cost?


A. depreciation of factory equipment
B. direct materials
C. supervisory salaries
D. total production costs

49. Variable costs per unit


A. can be estimated by the high-low method.
B. remains the same on a per unit basis when the level of activity changes.
C. are represented by the slope of the total cost line.
D. All of the above answers are correct.

50. Which of the following costs is least likely to be a variable cost?


A. sales commissions
B. direct labor
C. indirect materials
D. supervisory salaries

51. When the level of activity increases, total variable costs


A. decrease.
B. remain the same.
C. increases in direct proportion to the increase in activity.
D. increases, but at a slower rate than the level of activity.

52. When the level of activity increases, the variable cost per unit
A. decreases.
B. remains constant.
C. increases.
D. fluctuates, depending on the amount of the increase in activity.

53. When the level of activity increases, total fixed costs


A. decrease.
B. remain the same.
C. increase.
D. change, but the direction depends on the specific situation.

54. When the level of activity increases, the fixed cost per unit
A. decreases.
B. remains the same.
C. increases.
D. fluctuates, depending on the amount of the increase in activity.
Chapter 4 Cost-Volume-Profit Analysis 4-5

55. Which of the following components are included in a mixed cost?


A. a sunk cost and an opportunity cost
B. a fixed cost and a variable cost
C. a manufacturing cost and a selling cost
D. a product cost and a period cost

56. Three costs incurred by Hanson Company are summarized below at two different activity
levels, 500 and 800 units:

500 Units 800 Units


Cost A $12,000 $16,000
Cost B $24,000 $42,000
Cost C $17,500 $28,000

Which of these costs are variable?


A. A, B, and C
B. A and C
C. A only
D. C only

57. Within the relevant range, variable costs


A. are a different amount per unit at different activity levels.
B. are the same total amount at different activity levels.
C. are the same amount per unit at any activity level.
D. none of the above.

58. A cost is $3,600 at 1,000 units, $7,000 at 2,000 units, and $9,200 at 3,000 units. This cost
is a
A. variable cost.
B. fixed cost.
C. mixed cost.
D. step cost.

59. Step costs


A. are the same amount per unit for each range of volume.
B. are fixed for a range of volume but increase to a higher level when the upper
bound of the range is exceeded.
C. are a different total amount at every level of activity.
D. contain both a variable cost element and a fixed cost element.

60. A company has a cost that is $6.00 per unit at a volume of 9,000 units and $6.00 per unit
at a volume of 12,000 units. The cost is:
A. fixed
B. variable
C. mixed
D. incremental
4-6 Test Bank to accompany Jiambalvo Managerial Accounting, 4th Edition

61. Which of the following is not a method that is used to estimate variable and fixed costs?
A. account analysis
B. high low method
C. east-west method
D. regression analysis

62. Which of the following approaches to cost estimation is not based on fitting historical
data points to a line?
A. account analysis
B. high-low method
C. regression analysis
D. all are based on fitting historical points to a line

63. The account analysis approach to estimating fixed and variable costs
A. requires at least five years of historical data.
B. is based on the professional judgment of the manager.
C. is not useful for general and selling expenses.
D. is only used if the data for the high-low method or regression analysis is not
available.

64. When units produced and total production costs are graphed, the result is called a(n)
A. incremental analysis.
B. resource constraint.
C. contribution margin.
D. scattergraph.

65. The high-low method calculates the variable cost per unit as the
A. difference between fixed costs and total costs.
B. product of the number of units and the contribution margin per unit.
C. change in cost divided by the change in activity level for two points.
D. change in activity level divided by the change in cost for two points.

66. A significant weakness of the high-low method is that


A. a significant amount of management expertise is necessary to break out the
variable and fixed costs.
B. the two data points that are used may not be representative of the general relation
between cost and activity.
C. the calculations are so complex that a computer is usually necessary in order to
get accurate results.
D. monthly data must be collected for at least three years before the method can be
used.

67. Regression analysis


A. uses all the available data points to estimate a cost equation.
B. can be performed by many spreadsheet programs.
C. provides an equation that can be used to estimate total costs at different levels of
production.
D. All of the above are true.
Chapter 4 Cost-Volume-Profit Analysis 4-7

68. The range of activity for which estimates and predictions are likely to be accurate is the
A. incremental range.
B. margin of safety.
C. relevant range.
D. range of opportunity.

69. A cost is $50,000 at an activity level of 30,000 units, and $53,000 at an activity level of
33,000 units. The cost is:
A. fixed
B. variable
C. mixed
D. sunk

70. Duradyne, Inc. has total costs of $18,000 when 2,000 units are produced and $26,000
when 5,200 units are produced. During March, 4,000 units were produced and sold for $8
each. What is the variable cost per unit?
A. $2.50
B. $0.40
C. $2.00
D. $4.00

71. Clearance Depot has total monthly costs of $8,000 when 2,500 units are produced and
$12,400 when 5,000 units are produced. What is the estimated total monthly fixed cost?
A. $4,400
B. $6,580
C. $3,600
D. $8,800

72. ByteTown Computers has collected the following production data for the past four
months:
Units produced Total cost
7,000 $16,500
10,000 22,500
8,500 17,750
9,000 21,000

If the high-low method is used, what is the monthly total cost equation?
A. Total cost = $2,500 + ($2.00 × units produced)
B. Total cost = $3,750 + ($2.75 × units produced)
C. Total cost = $1,500 + ($ 2.17 × units produced)
D. Total cost = $500 + ($2.25 × units produced)
4-8 Test Bank to accompany Jiambalvo Managerial Accounting, 4th Edition

73. A regression analysis yields the following information:

Regression Statistics
Multiple R 0.961386
R Square 0.924262
Adjusted R Square 0.916688
Standard Error 39.10106
Observations 12

ANOVA
df SS MS F Significance F
Regression 1 186578 186578 122.0345 6.3E-07
Residual 10 15288.9 1528.89
Total 11 201867

Standard Lower Upper Lower Upper


Coefficients Error t Stat P-value 95% 95% 95% 95%
Intercept 955.01 225.526 4.2349 0.00173 452.579 1457.6 452.58 1457.6
X Variable 1 1.09 0.09912 11.0469 0.00634 0.87409 1.3158 0.8741 1.3158

What is the estimated cost for a production level of 1,200 units?


A. $1,308
B. $1,152
C. $2,263
D. More information is needed to determine the answer.

74. Total costs were $75,800 when 30,000 units were produced and $95,800 when 40,000
units were produced. Use the high-low method to find the estimated total costs for a
production level of 32,000 units.
A. $80,115
B. $76,000
C. $79,800
D. $91,800

75. Randall Sports has collected the following information over the last six months.

Month Units produced Total costs


March 10,000 $25,600
April 12,000 26,200
May 18,000 27,600
June 13,000 26,450
July 12,000 26,000
August 15,000 26,500

Using the high-low method, what is the variable cost per unit?
A. $0.22
B. $0.25
C. $2.00
D. $2.56
Chapter 4 Cost-Volume-Profit Analysis 4-9

Answers
48 B 72 A
49 D 73 C
50 D 74 C
51 C 75 B
52 B
53 B
54 A
55 B
56 D
57 C
58 C
59 B
60 B
61 C
62 A
63 B
64 D
65 C
66 B
67 D
68 C
69 C
70 A
71 C
CH APTE R 5
Marginal or Variable Costing
TRUE-FALSE

1. The cost of ending inventory using full costing is always greater than or equal to variable
costing inventory.

2. The cost of goods sold is always higher using varible costing than full costing.

3. Sales is higher using full costing than using variable costing if inventory levels are
increasing.

4. The units sold are higher using variable costing than using full costing.

5. If a company has no fixed costs, then variable costing income will equal full costing
income.

6. Variable costing income is more useful for decision making.

7. Variable costing is required for external reporting under generally accepted accounting
principles.

8. Full costing defers fixed costs of production into ending inventory.

9. The total selling and administrative expense is the same using variable and full costing.

10. In variable costing, fixed manufacturing overhead is considered a period cost.

11. Income statements of manufacturing firms prepared for external purposes use variable
costing.

12. Full costing ending inventory includes fixed and variable production costs.

13. Variable costing ending inventory includes variable production costs and variable selling
and administrative costs.

14. Contribution margin is reported on a variable costing income statement.

15. If sales equals production, then contribution margin will equal gross profit (margin).

16. Full costing income can be increased by increasing production without increasing sales.

17. When the number of units produced exceeds the number of units sold, variable costing
yields a higher income than full costing.

18. Variable costing income can be increased by increasing production without increasing sales.
5-2 Test Bank to accompany Jiambalvo Managerial Accounting 4th Edition

19. The inventory cost per unit under variable costing will be reduced if the number of units
produced increases.

20. When the number of units produced is less than the number sold, variable costing yields
higher income than full costing.

21. Full costing can give higher income than variable costing as long as inventory levels
continue to increase.

22. There are no cash flow consequences associated with increasing full costing income through
increasing production.

23. Just-in-time (JIT) inventory management systems cause a much larger difference between
variable costing income and full costing income.

24. The use of variable costing encourages management of earnings by adjusting production
volume.

25. Variable costing facilitates C-V-P analysis.

Answers

1 T 6 T 11 F 16 T 21 T
2 F 7 F 12 T 17 F 22 F
3 F 8 T 13 F 18 F 23 F
4 F 9 T 14 T 19 F 24 F
5 T 10 T 15 F 20 T 25 T
Chapter 5 Variable Costing 5-3

MULTIPLE CHOICE

26. Full costing


A. is the same as absorption costing.
B. considers fixed manufacturing overhead as part of the cost of inventory.
C. often does not provide the information needed for C-V-P analysis.
D. All of the above choices are correct.

27. Which of the following is treated differently in full costing than in variable costing?
A. Direct materials
B. Fixed manufacturing overhead
C. Direct labor
D. Variable manufacturing overhead

28. Which of the following is treated as a product cost in variable costing?


A. Sales commissions
B. Administrative salaries
C. Fixed manufacturing overhead
D. Direct labor

29. Which of the following is treated as a product cost in full costing?


A. Sales commissions
B. Administrative salaries
C. Security at the factory
D. Security at corporate headquarters

30. Full costing is


A. more useful for decision making.
B. required for financial reporting under generally accepted accounting principles.
C. better because income cannot be affected by increasing production.
D. All of the above.

31. In variable costing, when does fixed manufacturing overhead become an expense?
A. Never.
B. In the period when the product is sold.
C. In the period when the expense is incurred.
D. In the period when other expenses are at the lowest level.

32. In full costing, when does fixed manufacturing overhead become an expense?
A. In the period when other fixed costs are at the highest level.
B. In the period when the product is sold.
C. In the period when the expense is incurred.
D. When the controller decides that the expense should be recognized.
5-4 Test Bank to accompany Jiambalvo Managerial Accounting 4th Edition

33. In variable costing, which of the following would be included in inventory?


A. Fixed production cost
B. Variable selling cost
C. Fixed selling costs
D. none of the above items would be in inventory in variable costing

34. In full costing, which of the following would be included in inventory?


A. Fixed production cost
B. Variable selling cost
C. Fixed selling costs
D. None of the above items would be in inventory in full costing

35. Noth Company’s manufacturing costs for 2010 are as follows:

Direct materials $86,000


Direct labor $102,000
Depreciation of factory equipment $41,000
Other fixed manufacturing overhead $72,000

What amount should be considered product costs for external reporting purposes?
A. $260,000
B. $188,000
C. $229,000
D. $301,000

36. Train Company’s fixed manufacturing overhead costs totaled $220,000 and variable selling
costs totaled $190,000. Under full costing, how should these costs be classified?

Period Costs Product Costs


A. $190,000 $220,000
B. $410,000 $0
C. $0 $410,000
D. $220,000 $190,000

37. West Company’s manufacturing costs for 2010 are as follows:

Direct materials $100,000


Direct labor $250,000
Depreciation of factory equipment $30,000
Other fixed manufacturing overhead $50,000

What amount should be considered product costs for external reporting purposes?
A. $430,000
B. $380,000
C. $350,000
D. $400,000
Chapter 5 Variable Costing 5-5

38. Wilson Company’s fixed manufacturing overhead costs totaled $400,000 and variable
selling costs totaled $250,000. Under full costing, how should these costs be classified?

Period Costs Product Costs


A. $400,000 $250,000
B. $0 $650,000
C. $650,000 $0
D. $250,000 $400,000

39. Peak Manufacturing produces snow blowers. The selling price per snow blower is $100.
Costs involved in production are:

Direct Material per unit $20


Direct Labor per unit 12
Variable manufacturing overhead per unit 10
Fixed manufacturing overhead per year $148,500

In addition, the company has fixed selling and administrative costs of $150,000 per year.
During the year, Peak produces 45,000 snow blowers and sells 30,000 snow blowers.
Ignoring taxes, how much will full costing profit differ from variable costing profit?
A. $148,500
B. $49,500
C. $94,500
D. $74,250

40. Which of the following items appears on a variable costing income statement but not on a
full costing income statement?
A. Sales
B. Gross margin
C. Net income
D. Contribution margin

41. Variable costing income is a function of:


A. units sold only
B. units produced only
C. both units sold and units produced
D. neither units sold nor units produced

42. Which of the following items on a variable costing income statement will change in direct
proportion to a change in sales?
A. Sales, contribution margin, income.
B. Sales, variable costs, contribution margin.
C. Sales, variable costs, contribution margin, fixed costs and income.
D. Sales, variable costs, and fixed costs.
5-6 Test Bank to accompany Jiambalvo Managerial Accounting 4th Edition

43. If income is positive and fixed costs exist, which will increase or decrease at a greater rate
than sales changes on a variable costing income statement?
A. Variable costs
B. Fixed costs
C. Contribution margin
D. Income

44. Nilsan Company experienced the following costs in 2010:

Direct materials $2.25 / unit


Direct labor $4.10 / unit
Variable manufacturing overhead $0.75 / unit
Variable selling $3.00 / unit
Fixed manufacturing overhead $60,000
Fixed selling $35,000
Fixed administrative $20,000

During 2010 the company manufactured 120,000 units and sold 145,000 units. Assume the
same unit costs in all years. Total variable costs on the company’s 2010 contribution income
statement will be:
A. $1,464,500
B. $1,029,500
C. $1,102,000
D. $1,537,000

45. Washington Supply Company experienced the following costs in 2010:

Direct materials $3.50 / unit


Direct labor $2.55 / unit
Manufacturing Overhead Costs
Variable $1.50 / unit
Fixed $20,000
Selling & Administrative Costs
Variable selling $2.15 / unit
Fixed selling $8,000
Fixed administrative $7,000

During the year the company manufactured 95,000 units and sold 80,000 units. If the
average selling price per unit was $20, how much was the company’s contribution margin?
A. $996,000
B. $776,000
C. $824,000
D. $1,116,000
Chapter 5 Variable Costing 5-7

46. Spacet Excavating Company experienced the following costs in 2010:

Direct materials $1.75 / unit


Direct labor $2.00 / unit
Variable manufacturing overhead $2.50 / unit
Variable selling $.75 / unit
Fixed manufacturing overhead $50,000
Fixed selling $15,000
Fixed administrative $5,000

During the year the company manufactured 100,000 units and sold 80,000 units. If the
average selling price per unit was $22.65 what is the company’s contribution margin per
unit?
A. $16.40
B. $15.65
C. $18.90
D. $13.65

47. Data from Madison Company for 2010 is as follows:

Sales $20 / unit


Variable cost of goods sold ??
Fixed manufacturing overhead $85,000
Variable selling & administrative ??
Fixed selling & administrative $150,000

The company produced 145,000 units during the year and sold 130,000 units. Variable
production costs per unit and fixed costs have remained constant all year. Profit for the year
was $1,000,000. How much was the company’s contribution margin?
A. $765,000
B. $1,235,000
C. $1,365,000
D. Not enough information provided to determine the answer.
5-8 Test Bank to accompany Jiambalvo Managerial Accounting 4th Edition

48. During the past year, Cutt Company manufactured 25,000 units and sold 20,000 units.
Production costs during the year were as follows:

Fixed manufacturing overhead $550,000


Variable manufacturing overhead $380,000
Direct labor $278,000
Direct materials $214,000

Sales totaled $1,270,000, variable selling and administrative costs totaled $110,000, and
fixed selling and administrative costs totaled $170,000. There were no units in beginning
inventory. How much is the contribution margin per unit?
A. $6.62
B. $23.12
C. $28.62
D. $24.22

49. Peak Manufacturing produces snow blowers. The selling price per snow blower is $100.
Costs involved in production are:

Direct material per unit $20


Direct labor per unit 12
Variable manufacturing overhead per unit 10
Fixed manufacturing overhead per year $148,500

In addition, the company has fixed selling and administrative costs of $150,000 per year.
During the year, Peak produces 45,000 snow blowers and sells 30,000 snow blowers. What
is variable cost of goods sold?
A. $1,408,500
B. $1,260,000
C. $1,359,900
D. $2,038,500

50. Peak Manufacturing produces snow blowers. The selling price per snow blower is $100.
Costs involved in production are:

Direct material per unit $20


Direct labor per unit 12
Variable manufacturing overhead per unit 10
Fixed manufacturing overhead per year $148,500
Chapter 5 Variable Costing 5-9

In addition, the company has fixed selling and administrative costs of $150,000 per year.
During the year, Peak produces 45,000 snow blowers and sells 30,000 snow blowers. How
much is net income using variable costing?
A. $1,440,000
B. $1,740,000
C. $1,491,000
D. $1,441,500

51. The following information relates to Carmin Industries for fiscal 2011, the company’s first
year of operations:

Units produced 100,000


Units sold 80,000
Units in ending inventory 20,000
Fixed manufacturing overhead $650,000

How much fixed manufacturing overhead would be expensed in 2011 using variable
costing?
A. $520,000
B. $130,000
C. $650,000
D. $0

52. Lenat’s contribution income statement utilizing variable costing appears below:

Lenat Company
Income Statement
For the Year ended December 31, 2010
Sales ($28 / unit) $1,120,000
Less variable costs:
Cost of goods sold 560,000
Selling & administrative costs 96,000 656,000
Contribution margin 464,000
Less fixed costs:
Manufacturing overhead 80,000
Selling & administrative costs 90,000 170,000
Profit $294,000

Lenat Company produced 50,000 units during the year. Variable costs per unit and fixed
production costs have remained constant the entire year. There were no beginning
inventories. How much is the dollar value of the ending inventory using full costing?
A. $140,000
B. $160,000
C. $156,000
D. $128,000
5-10 Test Bank to accompany Jiambalvo Managerial Accounting 4th Edition

Multiple Choice Answers


26 D 41 C
27 B 42 B
28 D 43 D
29 C 44 A
30 B 45 C
31 C 46 B
32 B 47 B
33 D 48 B
34 A 49 B
35 D 50 D
36 A 51 C
37 A 52 C
38 D
39 B
40 D
Chapter 5 Variable Costing 5-11

EXERCISES

110. Bud Cooling Company is a small manufacturer of window air conditioners. The units sell
for $150 each. In 2011, the company produced 1,000 units and sold 800 units. Below are
variable and full costing income statements for 2011.

Bud Cooling Company


Variable Costing Income Statement
For the Year Ending December 31, 2011
Sales $120,000
Less variable costs:
Variable cost of goods sold $15,000
Variable selling expense 5,000 20,000
Contribution margin 100,000
Less fixed costs:
Fixed manufacturing expense 25,000
Fixed selling expense 10,000
Fixed administrative expense 15,000 50,000
Net income $50,000

Bud Cooling Company


Full Costing Income Statement
For the Year Ending December 31, 2011
Sales $120,000
Less cost of goods sold 35,000
Gross margin 85,000
Less selling and administrative expenses:
Selling expense $15,000
Administrative expense 15,000 30,000
Net income $55,000

Reconcile the difference in profit between the two income statements.

Answer
Fixed manufacturing overhead $25,000
Divided by units produced 1,000
Fixed manufacturing overhead per unit $ 25.00

Amount of fixed manufacturing overhead in ending inventory:


$25 × 200 units = $5,000
5-12 Test Bank to accompany Jiambalvo Managerial Accounting 4th Edition

111. The following information is available for Captain Spa, a manufacturer of above-ground spa
kits:
2011 2012 Total
Units produced 20,000 16,000 36,000
Units sold 18,000 18,000 36,000

Selling price per unit $8,000 $8,000


Direct material per unit $1,600 $1,600
Direct labor per unit $3,000 $3,000

Variable manufacturing overhead per unit $600 $600


Fixed manufacturing overhead per year $4,800,000 $4,800,000
Fixed selling and administrative expense per year$3,000,000 $3,000,000

In its first year of operations, the company produced 20,000 units, but was only able to sell
18,000 units. In its second year, the company needed to get rid of excess inventory (the
extra 2,000 units produced but not sold in 2011) so it cut back production to 16,000 units.

a. Calculate profit for both years using variable costing.


b. Does variable costing profit present a more realistic view of firm performance in the
two years? Explain.

Answer
a. 2011 2012 Total
Variable manufacturing costs per unit $5,200 $5,200

Sales ($8,000 × 18,000 units) $144,000,000 $144,000,000


Less cost of goods sold:
($5,200 × 18,000 units) 93,600,000 93,600,000
Contribution margin 50,400,000 50,400,000
Less fixed costs:
Manufacturing 4,800,000 4,800,000
Selling and administrative 3,000,000 3,000,000
Net income $42,600,000 $42,600,000 $85,200,000

Ending inventory ($5,200 × 2,000) $10,400,000 $0

b. Variable costing presents a more realistic view of firm performance in that income is
the same in both years which is consistent with the firm having the same cost
structure and level of sales in both years.
Chapter 5 Variable Costing 5-13

112. Raeon, Inc. produces car radios. The selling price per radio is $1,000. Costs involved in
production are:

Direct material $100


Direct labor 100
Variable manufacturing overhead 50
Fixed manufacturing overhead per year $250,000

In addition, the company has fixed selling and administrative costs:

Fixed selling costs per year $175,000


Fixed administrative costs per year $75,000

During the year, Raeon produces 1,000 car radios and sells 800 radios. What is the value of
ending inventory using variable costing?

Answer
Ending inventory under variable costing: $250 × 200 = $50,000

113. Raeon, Inc. produces car radios. The selling price per radio is $1,000. Costs involved in
production are:

Direct material $100


Direct labor 100
Variable manufacturing overhead 50
Fixed manufacturing overhead per year $250,000

In addition, the company has fixed selling and administrative costs:

Fixed selling costs per year $175,000


Fixed administrative costs per year $75,000

During the year, Raeon produces 1,000 car radios and sells 800 radios. How much is profit
using variable costing?

Answer
Sales ($1,000 × 800) $ 800,000
Less variable cost of goods sold ($250 × 800) 200,000
Contribution margin 600,000
Less Fixed manufacturing overhead $250,000
Selling expense 175,000
Administrative expense 75,000 500,000
Profit $100,000
5-14 Test Bank to accompany Jiambalvo Managerial Accounting 4th Edition

114. Raeon, Inc. produces car radios. The selling price per radio is $1,000. Costs involved in
production are:

Direct material $100


Direct labor 100
Variable manufacturing overhead 50
Fixed manufacturing overhead per year $250,000

In addition, the company has fixed selling and administrative costs:

Fixed selling costs per year $175,000


Fixed administrative costs per year $75,000

During the year, Raeon produces 1,000 car radios and sells 800 radios.
a. How much is profit using full costing?
b. How much fixed manufacturing overhead is in ending inventory under full costing?

Answer
a. VC: $1,000 - $250 = $750

$750 × 800 = $600,000


Less FC = $175,000 + $75,000 = 250,000
Profit = $ 350,000

b. $250 × 200 = $50,000


Chapter 5 Variable Costing 5-15

115. Below is a variable costing income statement for Wilner Glass Company, a maker of bottles
for the beverage industry. For the coming year, the company is considering hiring two
additional sales representatives at $80,000 each for base salary plus 5 percent of their sales
for commissions. The company anticipates that each sales representative will generate
$900,000 of incremental sales. The budget for 2011 follows:

Wilner Glass Company


Budgeted Variable Costing Income Statement
For the Year Ending December 31, 2011
Sales $15,000,000
Less variable costs:
Cost of goods sold $5,000,000
Selling expense 4,000,000 9,000,000
Contribution margin 6,000,000
Less fixed costs:
Manufacturing expense 2,300,000
Selling expense 1,200,000
Administrative expense 2,000,000 5,500,000
Net income $ 500,000

Calculate the impact on profit of the proposed hiring decision. Should the company hire the
two additional sales representatives?

Answer
Contribution margin ÷ sales = contribution margin ratio
$6,000,000 ÷ $15,000,000 = 0.40

(Incremental sales × CMR) – incremental salaries = incremental profit


($1,800,000 × .40) - $160,000 – ($1,800,000 × .05) = $470,000 profit increase

Since profit increases, Wilner Glass Company should hire the two additional sales
representatives.
5-16 Test Bank to accompany Jiambalvo Managerial Accounting 4th Edition

116. Superior Electronics produces a wireless home security device that allows consumers to
arm/disarm their security system from their cars. Information on the first three years of
business is as follows:

2011 2012 2013 Total


Units sold 20,000 20,000 20,000 60,000
Units produced 20,000 25,000 15,000 60,000
Fixed production costs $500,000 $500,000 $500,000
Variable production costs per unit $100 `$100 $100
Selling price per unit $200 $200 $200
Fixed selling and administrative
expense $150,000 $150,000 $150,000

a. Calculate profit and the value of ending inventory for each year using variable
costing.
b. Explain why, using variable costing, profit does not fluctuate from year to year.

Answer
a. 2011 2012 2013
Units sold 20,000 20,000 20,000
Selling price per unit $ 200 $ 200 $ 200
Sales $4,000,000 $4,000,000 $4,000,000
Less variable cost of goods sold:
($100 × 20,000) 2,000,000 2,000,000 2,000,000
Contribution margin 2,000,000 2,000,000 2,000,000
Less fixed costs:
Production 500,000 500,000 500,000
Selling and administrative 150,000 150,000 150,000
Profit $1,350,000 $1,350,000 $1,350,000

Ending inventory ($100 × 5,000) $0 $500,000 $0

c. Profit does not fluctuate each period. Fixed manufacturing overhead is treated as a
period cost and expensed each year even if units produced differ from the units sold.
Chapter 5 Variable Costing 5-17

117. The following information is available for Captain Spa, a manufacturer of above-ground spa
kits:
2011 2012 Total
Units produced 20,000 16,000 36,000
Units sold 18,000 18,000 36,000
Selling price per unit $8,000 $8,000
Direct material per unit $1,600 $1,600
Direct labor per unit $3,000 $3,000

Variable manufacturing overhead per unit $600 $600


Fixed manufacturing overhead per year $4,800,000 $4,800,000
Fixed selling and administrative expenseper year $3,000,000 $3,000,000

In its first year of operation, the company produced 20,000 units, but was only able to sell
18,000 units. In its second year, the company needed to get rid of excess inventory (the
extra 2,000 units produced but not sold in 2011) so it cut back production to 16,000 units.

Calculate profit for both years using full costing.

Answer
2011 2012
Fixed manufacturing overhead $4,800,000 $4,800,000
Divided by units produced 20,000 16,000
Fixed manufacturing overhead per unit 240 300
Variable manufacturing costs per unit 5,200 5,200
Full cost per unit $ 5,440 $ 5,500

Sales ($8,000 × 18,000 units) $144,000,000 $144,000,000


Less cost of goods sold:
($5,440 × 18,000) 97,920,000
($5,440 × 2,000 + $5,500 × 16,000) 98,880,000
Gross margin 46,080,000 45,120,000
Less selling and administrative expense 3,000,000 3,000,000
Net income $43,080,000 $42,120,000
5-18 Test Bank to accompany Jiambalvo Managerial Accounting 4th Edition

118. The following information relates to Ixim Production for fiscal year 2011, the company’s
first year of operations:

Units produced 20,000


Units sold 17,000
Selling price per unit $30
Direct material per unit $5
Direct labor per unit $5
Variable manufacturing overhead per unit $2
Variable selling cost per unit $3
Annual fixed manufacturing overhead $160,000
Annual fixed selling and administrative expense $80,000

a. Prepare an income statement using full costing.


b. Prepare an income statement using variable costing.

Answer
a.
Ixim Production
Full Costing Income Statement
For the Year Ending December 31, 2011
Sales ($30 × 17,000) $510,000
Less cost of goods sold ($20 × 17,000)* 340,000
Gross margin 170,000
Less selling and administrative expenses:
Fixed selling and administrative expense $80,000
Variable selling expenses ($3 × 17,000) 51,000 131,000
Profit $39,000

* Product cost per unit: $5 + $5 + $2 + $8 = $20

b.
Ixim Production
Variable Income Statement
For the Year Ending December 31, 2011
Sales ($30 × 17,000) $510,000
Less variable expenses
Production costs ($12 × 17,000) 204,000
Selling costs ($3 × 17,000) 51,000 255,000
Contribution margin 255,000
Less fixed expenses
Manufacturing overhead 160,000
Selling and administrative 80,000 240,000
Profit $15,000
Chapter 5 Variable Costing 5-19

119. A company has a variable manufacturing cost of $3.00 per unit, a variable selling cost of
$1.00 per unit; a fixed manufacturing cost of $100,000 per year; and a fixed selling and
administrative cost of $60,000 per year. The selling price is $9.00 per unit. During the year,
50,000 units are produced and 42,000 units are sold.
a. What is the cost per unit of inventory using variable costing?
b. What is the cost per unit of inventory using full costing?
c. Prepare an income statement without report titles using variable costing.
d. Prepare an income statement without report titles using full costing.

Answer:
a. $3.00

b. $3.00 + (100,000 / 50,000) = $5.00

c. Sales (42,000 × $9) $378,000


Variable Mfg Cost (42,000 × $3) $126,000
Variable Selling Cost (42000 × $1) 42,000 168,000
Contribution Margin $210,000
Fixed Production Costs 100,000
Fixed Selling & Admin Costs 60,000 $160,000
Net Income $ 50,000

d. Sales (42,000 × $9) $378,000


Cost of Goods Sold (42,000 × $5) 210,000
Gross Profit $168,000
Selling & Admin [$60,000 + ($1 × 42,000)] 102,000
Net Income $ 66,000

120. Raeon, Inc. produces car radios. The selling price per radio is $1,000. Costs involved in
production are:

Direct material $100


Direct labor 100
Variable manufacturing overhead 50
Fixed manufacturing overhead per year $250,000

In addition, the company has fixed selling and administrative costs:

Fixed selling costs per year $175,000


Fixed administrative costs per year $75,000

During the year, Raeon produces 1,000 car radios and sells 800 radios. What is the value of
ending inventory using full costing?
5-20 Test Bank to accompany Jiambalvo Managerial Accounting 4th Edition

Answer
Variable cost per unit $250
Fixed manufacturing overhead per unit
($250,000 ÷ 1,000 units) 250
Full cost per unit $500

Ending inventory under full costing: $500 × 200 units = $100,000

121. Raeon, Inc. produces car radios. The selling price per radio is $1,000. Costs involved in
production are:

Direct material $100


Direct labor 100
Variable manufacturing overhead 50
Fixed manufacturing overhead per year 250,000

In addition, the company has fixed selling and administrative costs:

Fixed selling costs per year $175,000


Fixed administrative costs per year 75,000

During the year, Raeon produces 1,000 car radios and sells 800 radios. How much is profit
using full costing?

Answer
Sales ($1,000 × 800 pairs) $800,000
Less: Cost of goods sold ($500 × 800 pairs) 400,000
Gross margin 400,000
Less: Selling expenses 175,000
Administrative expense 75,000 250,000
Profit $150,000

122. Superior Electronics produces a wireless home security device that allows consumers to
arm/disarm their security system from their cars. Information on the first three years of
business is as follows:

2011 2012 2013 Total


Units sold 20,000 20,000 20,000 60,000
Units produced 20,000 25,000 15,000 60,000
Fixed production costs $500,000 $500,000 $500,000
Variable production costs per unit $100 $100 $100
Selling price per unit $200 $200 $200
Fixed selling and administrative
expense $150,000 $150,000 $150,000
Chapter 5 Variable Costing 5-21

a. Calculate profit and the value of ending inventory for each year using full costing.
b. Explain why profit fluctuates from year to year even though the number of units
sold, the selling price, and the cost structure remain constant.

Answer
2011 2012 2013
Fixed production overhead $500,000 $500,000 $500,000
Divided by units produced 20,000 25,000 15,000
Fixed production overhead per unit $25.00 $20.00 $33.33
Variable production costs per unit 100.00 100.00 100.00
Full cost per unit $125.00 $120.00 $133.33

Sales ($200 × 20,000 units) $4,000,000 $4,000,000 $4,000,000


Less cost of goods sold:
($125 × 20,000) 2,500,000
($120 × 20,000) 2,400,000
($120 × 5,000) + ($133.33 × 15,000) 2,599,950

Gross margin 1,500,000 1,600,000 1,400,050


Less selling and admin expense 150,000 150,000 150,000
Net income $1,350,000 $1,450,000 $1,250,050

Ending inventory($120 × 5,000) $0 $600,000 $0

b. Even though sales revenue amounts are the same in each period, profit fluctuates. This
results because different quantities are produced each period which affects the fixed
manufacturing overhead in cost of goods sold versus ending inventory.

123. Last month, RainRunner produced 90,000 buckets and sold 85,000 of them at a price of $30
per bucket. Manufacturing costs consisted of direct materials of $200,000, direct labor of
$320,000, variable manufacturing overhead of $155,000 and fixed manufacturing overhead
of $396,000. General and administrative fixed costs totaled $60,000.

a. Calculate RainRunner’s net income using full costing.


b. Calculate RainRunner’s net income using variable costing.
5-22 Test Bank to accompany Jiambalvo Managerial Accounting 4th Edition

Answer
a. Revenue = 85,000 × $30 = $2,550,000

Cost of goods sold = ($7.5 × 85,000) + ($4.4 × 85,000) = $1,011,500

Net income = $2,550,000 – $1,011,500 – $60,000 = $1,478,500

b. Revenue = 85,000 × $30 = $2,550,000

Cost of goods sold = ($7.50 × 85,000) = $637,500

Net income = $2,550,000 – $637,500 – $396,000 – $60,000 = $1,456,500

124. A company has $8.00 per unit in variable production costs and $3.00 per unit in variable
selling and administrative costs. The annual fixed production cost is $180,000. The annual
fixed selling and administrative cost is $20,000.

a. Complete the table below for the number of units and dollar value of ending
inventory for each year. Assume a FIFO flow.

2010 2011 2012 2013


Units Produced 60,000 70,000 80,000 90,000
Units Sold 55,000 72,000 82,000 91,000
Units in ending inventory
Ending inventory using
variable costing
Ending inventory using
full costing

b. Assume that the selling price and cost structure stayed the same over the 4 year
period. How would the total income compare over the period between variable and
full costing?
Chapter 5 Variable Costing 5-23

Answer
a.
2010 2011 2012 2013
Units Produced 60,000 70,000 80,000 90,000
Units Sold 55,000 72,000 82,000 91,000
Units in ending inventory 5,000 3,000 1,000 0
Ending inventory using variable costing
$3 × 5,000 = $15,000
$3 × 3000 = $9,000
$3 × 1,000 = $3,000 $0
Ending inventory using full costing
$6 × 5,000 = $30,000
$5.57 × 3,000 = $16,710
($5.25 × 1,000) - $5,250 $0

b. Since sales equals production for the 4 year period, income would be the same.

SHORT-ANSWER ESSAYS

125. Explain the significant difference between variable costing and full costing.

Answer
The significant difference between variable costing and full costing is the treatment of fixed
manufacturing overhead. In variable costing, fixed manufacturing overhead is treated as a
period cost and expensed as it is incurred. In full costing, fixed manufacturing overhead is
considered a cost that becomes part of inventory and is not expensed until the goods are
sold.

126. Why is a variable costing income statement more useful for internal purposes?

Answer
The format separates fixed and variable costs facilitating cost-volume-profit analysis. Also,
it discourages over-production since managers cannot increase income by increasing
production.

127. Under full costing, how does increasing production increase income? Does this work under
variable costing? Why or why not?

Answer
Since fixed production costs are included in the unit cost using full costing, increasing
production will reduce unit costs. When these reduced costs are included in cost of goods
sold, income will be higher. Variable costing treats fixed production costs as a period cost,
and expenses the same amount regardless of production. Thus, income is unaffected by
increasing production.
5-24 Test Bank to accompany Jiambalvo Managerial Accounting 4th Edition

128. Can a company continue to increase income indefinitely by using full costing?

Answer

It is possible that a growing company can do this. The only way to do this is to produce
more than is sold in each year. This will result in a continuous buildup of inventory. For
most companies, that would not be a good strategy as it would lead to expensive cash
outflow for buildups of inventories along with all the associated carrying costs.

129. What is the implication of a company being JIT on the full costing versus variable costing
discussion?

Answer
JIT companies have very little inventory and so there is very little difference between full
and variable costing income.
CH APTE R 6
Cost Allocation and Activity-Based Costing
TRUE-FALSE

1. Indirect costs occur because resources are shared by more than cost object.

2. Cost allocation is the process of assigning direct costs to products.

3. Cost allocation methods that provide the most accurate full cost for financial reporting
also provide the most accurate information for making decisions within the company.

4. One of the reasons that companies allocate costs is to reduce the frivolous use of common
resources.

5. One of the reasons that companies allocate costs is to discourage managers from
exploring the use of externally provided services.

6. From a decision-making standpoint, the allocated cost should measure the sunk cost of
using a company resource.

7. Once the opportunity cost associated with a shared resource is determined, it is unlikely
to change.

8. In order to provide full cost information for external reporting purposes, indirect
production costs must be allocated to goods produced.

9. Cost-plus contracts guarantee that the supplier will be paid for production costs plus
some fixed amount or percentage of the cost.

10. The more costs that are allocated to a cost plus contract, the more the supplier will be
paid under the contract.

11. A cost objective is the product, service or department that will receive the allocated cost.

12. Large cost pools that contain many different kinds of costs are the most useful to
managers making decisions.

13. The allocation base selected should have a cause-and-effect relationship with the costs to
be allocated.

14. In the direct method of allocating costs, service department costs are allocated only to
other service departments, not to the production departments.

15. When service departments costs are allocated using actual costs and actual usage, the
charge one operating manager receives will depend on the usage of other managers.
6-2 Test Bank to accompany Jiambalvo Managerial Accounting, 4th Edition

16. Allocating actual service department costs allows the service departments to pass on the
costs of inefficiencies to the production departments.
Chapter 6 Cost Allocation and Activity-Based Costing 6-3

17. Managers should not be held responsible for controllable costs.

18. Allocating fixed costs on a per unit basis leads to better decision making.

19. Allocating fixed costs on a per unit basis will make the managers receiving the
allocations perceive the costs as variable.

20. ABC is less likely than traditional costing systems to undercost complex, low-volume
products.

21. ABC is always cost effective.

22. Under the ABC approach costs are assigned to products using a cost driver.

23. Activity-based costing and activity-based management are synonymous.

24. Activity-based management aims at improving the efficiency and effectiveness of


business processes.

25. Activity-based management uses benchmarking to compare the cost of an activity in one
organization to the cost for a similar activity in another organization

Answers
1 T 6 F 11 T 16 T 21 F
2 F 7 F 12 T 17 F 22 F
3 F 8 T 13 T 18 F 23 F
4 T 9 T 14 F 19 T 24 T
5 F 10 T 15 T 20 T 25 T
6-4 Test Bank to accompany Jiambalvo Managerial Accounting, 4th Edition

MULTIPLE CHOICE

29. Indirect costs occur when


A. resources are shared by more than one product or service.
B. costs cannot be directly traced to products or services.
C. multiple departments share a piece of equipment.
D. All of the above are correct.

30. The process of assigning indirect costs is called


A. directional association.
B. variable costing.
C. cost allocation.
D. joint costing.

31. Which of the following is not a reason that companies allocate costs?
A. to calculate the full cost of products for financial reporting purposes
B. to discourage managers from using external suppliers
C. to reduce the frivolous use of company resources
D. to provide information needed by managers to make appropriate decisions

32. Costs may be allocated to


A. products.
B. services.
C. departments.
D. any of the above.

33. From a decision-making standpoint, the allocated cost should measure the
A. sunk cost of the equipment involved.
B. variable costs of the goods purchased.
C. opportunity cost of using a company resource.
D. product cost of the goods produced.

34. If managers are not charged for centrally administered services, they are likely to
A. seek outside suppliers.
B. limit their frivolous use of these services.
C. consider the services as free.
D. evaluate and consider lower-cost alternatives for the services.

35. Full cost information


A. is required by GAAP for external reporting purposes.
B. does not require allocation of indirect costs.
C. provides managers with the most accurate information for making decisions.
D. treats all costs as fixed costs.
Chapter 6 Cost Allocation and Activity-Based Costing 6-5

36. A contract which specifies that the suppler will be paid for the cost of production as well
as some fixed amount or percentage of cost is called a(n)
A. approved overrun.
B. cost-plus contract.
C. allocation plan.
D. indirect cost budget.

37. Cost-plus contracts are common in which of the following industries?


A. manufactured home builders
B. soft drink bottlers
C. defense contractors
D. newspaper publishers

38. A major problem with cost-plus contracts is that they


A. are not acceptable under GAAP.
B. cause the supplier to take significant financial risks.
C. require the supplier to use variable costing.
D. create an incentive to allocate as much cost as possible to the goods produced
under the cost-plus contract.

39. Which of the following is not a step in the cost allocation process?
A. List the steps in the production process.
B. Select an allocation base to relate the cost pools to the cost objectives.
C. Form cost pools.
D. Identify the cost objectives.

40. The cost objective is the


A. reason for allocating the cost.
B. calculation based on budgeted amounts.
C. product, service, or department that is to receive the allocation.
D. maximum amount to be allocated to any single department.

41. The product, service, or department that is to receive the cost allocation is called the
A. cost-plus recipient.
B. cost object.
C. terminal.
D. pool for manufacturing overhead.

42. Which of the following is least likely to be a cost objective?


A. salaries such as those in the accounting and personnel departments
B. individual products such as spades and mowers
C. product lines such as loans and estate plans
D. departments such as assembly and finishing
6-6 Test Bank to accompany Jiambalvo Managerial Accounting, 4th Edition

43. A grouping of individual costs whose total is allocated using one allocation base is called
a
A. cost objective.
B. cost pool.
C. direct cost.
D. sunk cost.

44. A cost pool is


A. not necessary in cost-plus contracts.
B. useful when separating mixed costs into their fixed and variable components.
C. allocated using a single allocation base.
D. a method of allocating costs among service departments.

45. The thing for which a cost is being calculated is a(n)


A. cost object
B. cost pool
C. cost driver
D. sunk cost

46. The overriding concern in forming a cost pool is to ensure that


A. there are no variable costs in the cost pool.
B. the total amount in the cost pool is less than the direct costs for the product.
C. only costs which have been budgeted are included in the cost pool.
D. the costs in the pool are homogeneous or similar.

47. Which of the following statements about cost pools is not true?
A. The costs in each of the cost pools should be homogeneous or similar.
B. Managers must make a cost-benefit decision when determining how many cost
pools are appropriate.
C. Only four different kinds of costs may be included in a single cost pool.
D. More cost pools usually provide more accurate information, but are more
expensive.

48. An allocation base


A. is the minimum amount to be allocated to a cost object.
B. coordinates the manufacturing overhead costs as they are incurred.
C. will always be less than the variable costs for a product.
D. relates the cost pool to the cost objectives.

49. An allocation base


A. relates the cost pool to the cost objectives.
B. is a characteristic that is common to all of the cost objectives.
C. ideally uses a cause-and-effect relationship.
D. All of the above are true.
Chapter 6 Cost Allocation and Activity-Based Costing 6-7

50. Which of the following is not a criterion used to allocate fixed costs?
A. ability to bear costs
B. equity
C. feasible outcomes
D. relative benefits

51. Which of the following is not a service department in a typical manufacturing firm?
A. security
B. fabrication
C. maintenance
D. personnel

52. The method of allocation which allocates service department costs to production
departments but not to other service departments is called the
A. equity method.
B. direct method.
C. reciprocal method.
D. sequential or step method.

53. Service department costs are allocated to producing departments


A. so that the costs can be allocated to the products in the producing departments.
B. because the costs of the service is not material.
C. so the service will not be purchased externally.
D. All of the above.

54. Which of the following allocations would not occur when the direct method is used in a
manufacturing company?
A. personnel department costs allocated to the maintenance department.
B. maintenance department costs allocated to the finishing department.
C. assembly department costs allocated to Product C.
D. All of the above allocations could occur when the direct method is used.

55. The advantage of allocating budgeted rather than actual service department costs is that
A. managers are not motivated to evaluate the charges.
B. only one cost pool is necessary.
C. service departments cannot pass on the costs of inefficiencies and waste.
D. this practice is acceptable under GAAP.

56. The type of costs which are affected by the manager’s decisions and for which the
manager should be held accountable are
A. indirect costs.
B. controllable costs.
C. basis costs.
D. pooled costs.
6-8 Test Bank to accompany Jiambalvo Managerial Accounting, 4th Edition

57. Managers are correct when they perceive that almost all cost allocations are
A. insignificant.
B. arbitrary.
C. designed to make them look bad.
D. unnecessary.

58. When fixed costs are unitized, they


A. are stated on a per unit basis.
B. may appear to be variable costs.
C. may cause managers to make decisions that are not in the best interest of the
company as a whole.
D. All of the above are true.

59. An allocation of a predetermined amount that is not affected by changes in the activity
level of the organizational unit receiving the allocation is called a(n)
A. allocation base.
B. unitized cost.
C. lump-sum allocation.
D. cost driver.

60. Lump-sum allocations


A. allocate fixed costs so that they appear fixed to the manager.
B. are not affected by the activity level in the department receiving the allocation.
C. Both A and B are true.
D. Neither A nor B is true.

61. Lump sum allocations


A. should generally be adjusted each month.
B. will change when the activity levels of any of the user departments change.
C. are not impacted by the usage of the allocated resource by other departments.
D. make fixed costs appear variable.

62. Which of the following is not a problem caused by assigning actual service department
costs to operating departments based on actual usage of service department activities by
the operating departments?
A. The cost assigned to one manager will be affected by the service usage of another
manager.
B. Inefficiencies in the service department will be passed along to the operating
departments.
C. Operating managers having high peak capacity requirements will not have to bear
the full cost of meeting this peak capacity.
D. All of the above are problems caused by actual allocation.
Chapter 6 Cost Allocation and Activity-Based Costing 6-9

63. The Copy Department of the Carnival Company is budgeted to incur $40,000 per month
in fixed costs and $0.02 per copy in variable costs. It allocates copy costs to user
departments as follows: Fixed costs are allocated (as a lump sum) based on budgeted
fixed costs and estimated peak demand for each department. Variable costs are allocated
based on the budgeted rate per copy times the department's actual usage. Which of the
following is not an advantage of this allocation scheme over allocating actual costs based
on actual usage?
A. Using departments are not charged for cost overruns in the copy department.
B. The amount charged to one using department is not affected by the number of
copies used by another department.
C. Managers in the using departments pay for the fixed costs that are created by their
demands for capacity.
D. All of the above are advantages of this allocation system.

64. Which of the following are desirable characteristics of an allocation received by a


manager of a producing department from a service department for using the resources of
the service department?
A. The amount of the allocation should be based solely on the usage of the service by
the producing department and not a function of the use of the service by other
departments.
B. The manager should be able to budget for the cost.
C. The allocation should force the production manager to pay for the capacity
demands the production manager is creating.
D. All of the above are desirable characteristics of an allocation received by a
manger of a producing department from a service department.

65. Companies which use only one or two cost pools rather than several cost pools
A. may have seriously distorted product costs.
B. will have higher record-keeping costs.
C. will be able to more accurately price products to cover the cost and generate a
profit.
D. are likely to be using ABC.

66. When activity based costing is implemented, the initial outcome is normally that:
A. the cost of all products will be higher.
B. The cost of all products will be lower
C. The cost of low volume products will be higher and the cost of high volume
products will be lower
D. The cost of low volume products will be lower and the cost of high volume
products will be higher.
6-10 Test Bank to accompany Jiambalvo Managerial Accounting, 4th Edition

Answers
29 D 51 B
30 C 52 B
31 B 53 A
32 D 54 A
33 C 55 C
34 C 56 B
35 A 57 B
36 B 58 D
37 C 59 C
38 D 60 C
39 A 61 C
40 C 62 D
41 B 63 D
42 A 64 D
43 B 65 A
44 C 66 C
45 A
46 D
47 C
48 D
49 D
50 C
Chapter 6 Cost Allocation and Activity-Based Costing 6-11

MATCHING

152. Match each of the following terms with the phrase that most closely describes it. Each
answer may be used only once.

_____ 1. ability to bear costs

_____ 2. activity-based costing

_____ 3. activity-based management

_____ 4. cost allocation

_____ 5. cost driver

_____ 6. cost objective

_____ 7. cost pool

_____ 8. cost-plus contract

_____ 9. lump-sum allocation

_____ 10. unitized costs

A. Assigning indirect costs to cost objectives


B. A method of allocation that assigns more costs to more profitable cost objects
C. A measure of the activity that is used to allocate costs
D. Fixed costs stated on a per unit basis
E. Price for this includes actual costs plus a fixed amount or percentage
F. A recently developed method of cost allocation that uses cost drivers to allocate costs to
products
G. The object or recipient of the cost allocation
H. Has the goal of improving efficiency and effectiveness by analyzing and costing activities
I. A grouping of individual costs whose total is allocated using one allocation base
J. Allocations of fixed costs in which predetermined amounts are allocated regardless of
changes in the level of activity

Answers
1. B 6. G
2. F 7. I
3. H 8. E
4. A 9. J
5. C 10. D
6-12 Test Bank to accompany Jiambalvo Managerial Accounting, 4th Edition

EXERCISES

153. Keystone Company’s copy department, which does almost all of the photocopying for the
sales department and the administrative department, budgets the following costs for the
year, based on the expected activity of 4,000,000 copies:

Salaries (fixed) $90,000


Employee benefits (fixed) 10,000
Depreciation of copy machines (fixed) 10,000
Utilities (fixed) 5,000
Paper (variable, 1 cent per copy) 40,000
Toner (variable, 1 cent per copy) 40,000

The costs are assigned to two cost pools, one for fixed and one for variable costs. The
costs are then assigned to the sales department and the administrative department. Fixed
costs are assigned on a lump-sum basis, 30 percent to sales and 70 percent to
administration. The variable costs are assigned at a rate of 3 cents per copy. Assuming
4,800,000 copies were made during the year, 2,500,000 for Sales and 2,300,000 for
Administration, calculate the copy department costs allocated to sales and separately to
Administration.

Answer
Total fixed costs to be allocated = $90,000 + 10,000 + 10,000 + 5,000 = $115,000.

Fixed costs allocated to Sales ($115,000  .3) $ 34,500


Variable costs allocated to Sales (2,500,000 copies  $.02) 50,000
Total costs allocated to Sales $ 84,500

Fixed costs allocated to Admin. ($115,000  .7) $ 80,500


Variable costs allocated to Admin (2,300,000 copies  $.02) 46,000
Total costs allocated to Sales $126,500
Chapter 6 Cost Allocation and Activity-Based Costing 6-13

154. Flowers Banking and Loan Company has a graphic design department that designs loan
forms and other documents used by the company’s two subsidiaries (Flowers Personal
Banking and Auburn Business Banking). For practical purposes, the costs of the graphic
design department are primarily fixed and relate to the salaries of the department’s two
employees.
Each subsidiary receives a cost allocation of $50 per design hour. Jobs requested by the
subsidiaries generally take weeks to complete often causing the subsidiaries to go outside
the company for design services rather than wait for jobs to be completed. Outside design
services cost $70 per hour. How does the allocation ($50 per design hour) compare to the
opportunity cost of using design services?

Answer
The allocation of $50 per hour must be less than the opportunity cost (subsidiaries are
willing to pay $70 per hour to avoid delays). Thus, they must have a benefit that exceeds
$70.
155. Tasco Manufacturing produces small electric motors used by appliance manufacturers. In
the past year, the company has experienced severe excess capacity due to competition
from a foreign company that has entered Tasco’s market. The company is currently
bidding on a potential order from Corinth Appliances for 6,000 Model 209 motors. The
estimated cost of each motor as follows:

Direct material $ 30
Direct labor 20
Overhead 80
Total $130

The predetermined overhead rate is $4 per direct labor dollar based on estimated annual
overhead of $12,000,000 and estimated annual direct labor of $3,000,000. The
$12,000,000 of overhead is composed of $6,000,000 of variable costs and $6,000,000 of
fixed costs. The largest fixed cost relates to depreciation of plant and equipment.

a. With respect to overhead, what is the opportunity cost of producing a Model 209
motor?
b. Suppose Tasco can win the Corinth business by bidding a price of $92 per motor
(but no higher price will result in a winning bid). Should Tasco bid $92?
c. Discuss how an allocation of overhead based on opportunity cost would facilitate
an appropriate bidding decision.

Answer
a. Direct material $30
Direct labor 20
Variable overhead ($2.00 per direct labor $)* 40
Total $90
* Variable overhead rate = $6,000,000 ÷ $3,000,000
6-14 Test Bank to accompany Jiambalvo Managerial Accounting, 4th Edition

b. Any bid greater than $90 (that’s accepted) will generate incremental profit. If
Tasco can get the order with a bid of $92, the company should bid this amount.

c. The opportunity cost related to overhead, in this case, is the variable overhead
amount. An allocation based on the opportunity cost idea, helps managers focus
on incremental costs—the information needed for decision making.

156. Precise Instrumentation manufactures a variety of electronic instruments that are used in
military and civilian applications. Sales to the military are generally on a cost-plus profit
basis with profit equal to 10 percent of cost. Instruments used in military applications
require more direct labor time because “fail-safe” devices must be installed.
At the start of the year, Precise estimates that the company will incur $50,000,000 of
overhead, $8,000,000 of direct labor, and 250,000 machine hours. Consider the Model
VR12 gauge that is produced for both civilian and military uses:

Civilian Military
Direct material $3,000 $3,500
Direct labor $ 900 $1,200
Machine hours 42 45

a. Calculate the cost of civilian and military versions of Model VR12 using both
direct labor dollars and machine hours as alternative allocation bases.
b. Explain why Precise Instruments may decide to use machine hours as an overhead
allocation base.

Answer
a. Overhead rate based on direct-labor dollars
($50,000,000 ÷ $8,000,000 labor) = $6.25 per dollar of labor

Overhead rate based on machine hours


($50,000,000 ÷ 250,000 machine hours) = $200 per machine hour

Civilian Military
Labor $ Mach.Hrs. Labor $ Mach.Hrs.
Direct material $3,000 $ 3,000 $ 3,500 $ 3,500
Direct labor 900 900 1,200 1,200
Overhead 5,625 8,400 7,500 9,000
Cost $9,525 $12,300 $12,200 $13,700

b. The price charged for the civilian version of the Model VR12 does not depend on
allocated costs. However, the military version is sold for “cost” plus 10 percent of
cost. Therefore, the company has an incentive to make cost appear higher rather
than lower. This can be accomplished by allocating overhead cost using machine
hours as the allocation base. This base results in a higher cost ($13,700) compared
to an allocation based on direct labor dollars which results in a cost of only
$12,200.
Chapter 6 Cost Allocation and Activity-Based Costing 6-15

157. Mooring Company allocates manufacturing overhead costs to products based on the
machine hours used. Manufacturing overhead costs are expected to total $166,950 in the
coming period and the company plans to use 47,700 machine hours in the same period.
Product X requires three machine hours per unit and Product Y requires four machine
hours per unit. The current production schedule calls for 7,500 units of Product X and
6,300 units of Product Y.

A. What is the overhead application rate per machine hour?


B. If production and overhead cost occur as scheduled, how much manufacturing
overhead will be allocated to Product X and how much will be allocated to
Product Y?

Answer
A. $166,950 / 47,700 = $3.50 per machine hour

B. Product X = 7,500 units × 3 machine hours/unit × $3.50 per machine hour =


$78,750

Product Y = 6,300 units × 4 machine hours/unit × $3.50 per machine hour = $88,200

158. Kendrick Apparel is the designer and maker of elaborate prom dresses. The president of
Kendrick wants to switch to an activity-based approach in the upcoming year to assign
prices to the gowns. Production line setups are a major activity at Kendrick. Next year
Kendrick expects to perform 1,500 setups at a total cost of $102,000. Kendrick plans to
produce 170 dresses of the debonair design, which will require three setups. How much
setup cost will be allocated to each dress of the debonair design that is produced?

Answer
Cost per setup = $102,000 ÷ 1,500 = $68

Cost of setups related to the A128 design = 3  $68 = $204

Setup cost per debonair dress = $204 ÷ 170 = $1.20

159. Cortez Company has three service departments (A1, A2, A3) and two production
departments (B1, B2). The following data relate to Cortez’s allocation of service
department costs:
Budgeted Costs Number of Employees
A1 $3,000,000 80
A2 2,000,000 60
A3 1,000,000 30
B1 300
B2 500
6-16 Test Bank to accompany Jiambalvo Managerial Accounting, 4th Edition

Service department costs are allocated by the direct method. The number of employees is
used as the allocation base for all service department costs.
a. Allocate service department costs to production departments.
b. Calculate the total service department cost allocated to each production
department.

Answer
B1 has 300 ÷ 800 = 37.5% of production department employees.

B2 has 500 ÷ 800 = 62.5% of production department employees.

Cost Allocated to
Department Costs B1 B2
A1 $3,000,000 $1,125,000 $1,875,000
S2 2,000,000 750,000 1,250,000
A3 1,000,000 375,000 625,000
Total cost $6,000,000 $2,250,000 $3,750,000

160. Dillard Industries produces electronic equipment for the marine industry. Maintenance
costs are allocated to assembly and testing on the basis of square footage occupied, and
computing costs are allocated on the basis of the number of computer terminals. The
following data relate to allocations of service department costs:
Maintenance Computing Assembly Testing
Service department costs $400,000 $800,000
Square footage 77,000 33,000
Terminals 6 10

Allocate the service department costs to production departments using the direct method.

Answer
Assembly Testing
Maintenance ($400,000) 70% 30%
Computing ($800,000) 37.5% 62.5%

Service dept. costs allocated to Assembly:


(.70  $400,000) + (.375  $800,000) = $280,000 + $300,000 = $580,000
Service dept. costs allocated to Testing:
(.30  $400,000) + (.625  $800,000) = $120,000 + $500,000 = $620,000
Chapter 6 Cost Allocation and Activity-Based Costing 6-17

161. Farm Land Financial Services has two divisions: Financial Planning and Business
Consulting. The firm’s accountants are in the process of selecting an allocation base to
allocate centrally provided personnel costs to the divisions. Two allocation bases have
been proposed—salary and headcount (number of employees). Personnel costs are
expected to be $2,500,000. The following data relate to the allocation:

Financial Planning Business Consulting


Salaries $14,000,000 $18,000,000
Headcount 150 50

a. Prepare a schedule showing the allocations to the two divisions using each
allocation base.
b. Referring to your answer to part a, explain why allocations are sometimes
considered arbitrary.

Answer
a. Allocation Base
Financial Planning Business Consulting
Proportion Amount Proportion Amount
Salaries .4375 $1,093,750 .5625 $1,406,250
Headcount .75 $1,875,000 .25 $625,000

b. Both headcount and salary appear to be plausible allocation bases, but they result
in very different allocations. This suggests that in many cases allocations are
somewhat arbitrary.

162. The costs of the personnel department at Wanson Company total $180,000. These costs
are allocated based on the number of employees in the production departments. If 66 of
Celtic’s 165 production employees work in the sanding department, what amount of the
personnel department costs should be allocated to the sanding department?

Answer
($180,000 / 165) × 66 = $72,000

163. The personnel department at Harton Company has $45,000 in budgeted costs for the
coming period. Harton is trying to determine whether to allocate these costs to the two
production departments based on the number of employees or on machine hours used in
the department. Information about the production departments is given below:

Department C Department D
Number of employees 15 35
Anticipated machine hours 600 400

Calculate the costs allocated to each of the production departments using each allocation
base. Comment on which allocation base is preferable.
6-18 Test Bank to accompany Jiambalvo Managerial Accounting, 4th Edition

Answer
When number of employees as the allocation base:
Department C = $45,000 × (15/50) = $13,500
Department D = $45,000 × (35/50) = $31,500

When number of machine hours as the allocation base:


Department C = $45,000 × (600/1,000) = $27,000
Department D = $45,000 × (400/1,000) = $18,000

The number of employees is a better allocation base because it is more closely related to
personnel department costs.

164. King Company has a regional division and a national division and a travel department
which supports the employees in both divisions. The fixed costs of the travel department
($30,000 per month) are allocated based on the peak usage of reservation services. The
national division has a peak monthly usage of 300 reservations and the regional has a
peak usage of 200 reservations. The variable costs of the travel department are allocated
to the divisions based on the number of reservations made, at a rate of $20 per
reservation.

A. If the regional division requires 180 reservations and the national division
requires 170 reservations, calculate the amount of travel department costs that will
be allocated to each of the divisions.
B. Explain why the allocation is higher to national when it uses fewer reservations.

Answer
A. Regional division = [$30,000 × (200/500)] + ($20 × 180) = $15,600

National division = [$30,000 × (300/500)] + ($20 × 170) = $21,400

B. National has higher peak needs and so is being charged a higher amount for
capacity.

165. Corporation has three departments, Civil, Criminal, Auditing and Probate. Each
department uses the services of the photocopying department. The photocopying
department has 10 copiers, and each copier can produce 100,000 copies per month and
has a fixed cost of $1,000 per month. The variable cost to produce a copy is $0.03 per
copy.

The three production departments have monthly consumption information as follows:

Peak Demand Average Demand March Usage


Civil 600,000 copies 300,000 410,000
Criminal 200,000 copies 180,000 190,000
Probate 200,000 copies 180,000 170,000
Chapter 6 Cost Allocation and Activity-Based Costing 6-19

During March, the actual costs incurred in the Copying Department were:
$10,300 fixed and $23,870 in variable costs.

A. Describe three alternative ways that the production departments could be charged
for services.
B. Assign the actual cost of March copying to the production departments based on
the actual copies used by each department. What is the cost per copy? (Round to
two decimal places for cost per copy and whole dollars for charges to
departments.)
C. Assign the cost of March copying to the production departments using a dual rate.
Assign the budgeted fixed cost based on peak usage requirements. Assign the
budgeted variable cost based on actual usage. What is the total cost per copy
assigned to each department?
D. Which way do you believe is more equitable? Why?

Answer
A. Actual cost per copy on actual copies used
Budgeted total cost per copy based on average usage times actual copies used.
Dual allocation as in c below

B. 4.44 cents per copy


Civil - $18,204
Criminal - $8,436
Probate $7,548

C. Civil: 4.46 cents per copy


(600,000/1,000,000 × 10,000) + (410,000 × .03) =$18,300

Criminal: 4.05 cents per copy


(200,000/1,000,000 × 10,000) + (190,000 × .03) = $7,700

Probate: 4.18 cents per copy


(200,000/1,000,000 × 10,000) + (170,000 × .03) = $7,100

D. Dual rate (c) will encourage the departments demanding the capacity to pay
capacity costs.

166. Metal Inc. received an offer from a retail company to purchase 2,000 metal outdoor tables
for $155 each. Metal Inc.’s accountants determine that the following costs apply to the
tables:

Direct material $ 90
Direct labor 45
Manufacturing overhead 70
Total $205
6-20 Test Bank to accompany Jiambalvo Managerial Accounting, 4th Edition

Of the $70 of overhead, $14 is variable and $56 relates to fixed costs. The $56 of fixed
overhead is allocated as $1.25 per direct labor dollar.
a. What will be the real effect on profit if the order is accepted?
b. Explain why managers who focus on reported cost per unit may be inclined to
turn down the order.

Answer
a. The manufacturing overhead allocation includes $56 of fixed cost which will not
increase if the special order is accepted (i.e., it is not an incremental cost). The
incremental revenue and incremental costs associated with the order suggests that
the company will be better off by $12,000 if the order is accepted.

Incremental revenue (2,000  $155) $310,000


Incremental costs
Material (2,000  $90) $180,000
Labor (2,000  $45) 90,000
Variable overhead (2,000  $14) 28,000 298,000
Incremental profit $ 12,000

b. Managers who focus on reported cost may (incorrectly) treat the $56 of fixed cost
as an incremental cost. In this case they will (incorrectly) conclude that the order
should not be accepted because the total cost of $205 is more than the offer price
of $155.

167. Clark Manufacturing allocates factory overhead using one cost pool with direct labor
hours as the allocation base. Clark has two production departments (A1 and A2). The
new accountant at Clark estimates that next year the total factory overhead costs will be
$4,000,000 and approximately 500,000 direct labor hours will be worked. The accountant
also estimates that A1 will use 150,000 direct labor hours and there will be about
$2,000,000 in overhead costs in A1. A2 will use 350,000 direct labor hours and there will
be $2,000,000 in overhead costs in A2. Clark has two products: R4 and R5. It takes two
direct labor hours in A1 and three direct labor hours in A2 to complete one unit of R4. It
takes one direct labor hour in A1 and four direct labor hours in A2 to complete one unit
of R5.

Which product will be undercosted and which will be overcosted using a single cost pool
system? Support your answer with appropriate calculations.
Chapter 6 Cost Allocation and Activity-Based Costing 6-21

Answer
The use of a single cost pool causes R4 to be undercosted and R5 to be overcosted. With
a single cost pool, both products receive the same allocation of cost per labor hour.
However, R4 uses relatively more time in A1 where overhead costs are high while R5
uses relatively more time in A2 where overhead costs are low.

The one-cost pool overhead rate is:


$4,000,000 ÷ 500,000 DLH = $8 /DLH

Each product requires 5 direct labor hours in total. Therefore, each will be allocated $40
in overhead costs ($8  5)

When separate cost pools are used, the amount of overhead allocated to each product if
Clark used a separate overhead cost pool for each production department is:

A1’s overhead rate will be $2,000,000 ÷ 150,000 DLH = $13.33 per direct labor hour
A2’s overhead rate will be $2,000,000 ÷ 350,000 DLH = $5.71 per direct labor hour

Overhead allocated to each unit of R4 will be:


(2 labor hours  $13.33) + (3 labor hours  $5.71) $43.79

Overhead allocated to each unit of R5 will be:


(1 labor hour  $13.33) + (4 labor hours  $5.71) $36.17

168. The following are six cost pools established for a company using activity-based costing.
The pools are related to the company’s products using cost drivers. For each of the
preceding cost pools, identify a possible cost driver.

COST POOL COST DRIVER


(1) Inspection of raw materials ______________
(2) Production equipment repairs and maintenance ______________
(3) Raw materials storage ______________
(4) Plant heat, light, water, and power ______________
(5) Finished product quality control ______________
(6) Production line setups ______________

Answer
COST POOL COST DRIVER
(1) Inspection of raw materials Number of receipts, pounds
(2) Production equipment repairs and maintenance Machine hours, square feet
(3) Raw materials storage Dollar value of raw materials
(4) Plant heat, light, water, and power Square footage
(5) Finished product quality control Number of inspections
(6) Production line setups Number of setups
6-22 Test Bank to accompany Jiambalvo Managerial Accounting, 4th Edition

169. Ralston’s Machining produces specialized equipment. Currently, overhead costs are
allocated at a rate of $5 per machine hour produced and the company used 20,000
machine hours last year. Ralston’s CEO has heard about ABC, and would like to see if it
makes any difference in the costs allocated to jobs at the company.

The accounting staff has provided the following information about manufacturing
overhead:
Amount Cost Driver
Setups $30,000 Number of setups
Equipment 20,000 Number of machine hours
Inspection 50,000 Number of inspections

The company estimates that it will perform 150 setups and 1,000 inspections each year
and will use 2,000 machine hours. Job CRT will require 18 setups, 85 machine hours, and
60 inspections.

A. Using ABC, what amount of manufacturing overhead will be allocated to Job


CRT?
B. What amount would Ralston allocate to job CRT using their current, traditional
system?
C. Why do the two methods yield such different answers?

Answer
A. Using ABC costing,
[18 × ($30,000/150)] + [85 × ($20,000/2,000)] + [60 × ($50,000/1,000)] = $7,450

B. Using the current system, 85 × ($100,000/2,000) = $4,250

C. The order uses the 4% of annual machine hours, but 12% of annual setup and 6%
of annual inspections. It is a heavy user of non-unit based drivers, and is costed
much higher when this is recognized.

170. Williams Appliances supplies parts for laundry and kitchen appliances. Customer orders
are placed over the Internet and are generally filled in one or two days using express mail
services.
Frank Farnsworth, a consultant with Consultancy Services, has been asked to conduct an
ABM study of inventory management at Williams Appliances. In this regard he has
determined that the cost of filling customer orders in the past year consisted primarily of
$250,000 of salary expense related to five workers who “pick” parts from the warehouse
and $550,000 of salary expense related to six workers who pack the orders for shipment.
In the past year, the company filled 100,000 orders. Based on work performed for a chain
of auto supply stores, Frank has determined a benchmark cost of $4 per order.

a. Comment on the advisability of comparing the costs at Williams Appliances to


those at an auto supply chain store.
Chapter 6 Cost Allocation and Activity-Based Costing 6-23

b. Frank has observed the following: Workers go to a box that contains individual
customer order sheets. They take the bottom order (the “oldest”) and go into the
warehouse with a handcart and a box. They then fill the order and carry the parts
to a packing station. Can you suggest ways of improving this process?

Answer
a. The cost of filling orders at Williams Appliances is:
($250,000 + $550,000) ÷ 100,000 = $8 per order
The cost of filling orders at the auto supply chain is only $4 per order. While this
is lower, it may be that, due to its size, the appliance auto chain has a “state of the
art” warehouse. It may be unrealistic for Williams Appliances, which is relatively
small, to compare itself to such a large company.

b. Possibly, order “pickers” can take multiple order sheets out to the warehouse
when individual orders are small. This will save considerable time going back and
forth to the warehouse and may lead to lower costs if the company is willing to
fire or reassign one or more of the five workers who pick parts.

SHORT-ANSWER ESSAYS

171. List the four major reasons that companies allocate costs.

Answer
Companies allocate costs (1) to provide information needed to make appropriate
decisions, (2) to reduce the frivolous use of common resources, (3) to encourage
managers to evaluate the efficiency of internally provided services, and (4) to calculate
the “full cost” of products for financial reporting purposes and for determining cost-based
prices.

172. What is a cost-plus contract? Why are cost-plus contracts used?

Answer
The supplier is paid for all costs of production as well as some fixed amount or
percentage of cost in a cost-plus contract. Cost-plus contracts are used when suppliers
need to be assured that they will be reimbursed for their costs and will not bear any of the
financial risk associated with the project.

173. What is the difference between a cost pool and a cost objective?

Answer
A cost pool is a grouping of individual costs whose total is allocated using one allocation
base. A cost objective is the product, service, or department that will receive the cost
allocation.
6-24 Test Bank to accompany Jiambalvo Managerial Accounting, 4th Edition

174. Why is it better to allocate budgeted rather than actual service department costs?

Answer
If budgeted costs are allocated, service departments cannot pass on the costs of
inefficiencies and waste to the user departments. If actual service department costs are
allocated, the service departments have no incentive to control their costs. Budgeted
allocations can also be more timely.

175. Why are lump-sum allocations used to allocate fixed costs?

Answer
When lump-sum allocations are used to allocate fixed costs, the costs will appear fixed to
the managers whose departments receive the allocations. If lump-sum allocations are not
used, an increase in volume will lead to an increased allocation of fixed costs and may
cause incorrect decisions.

176. If a manager is allocated the costs of service departments based on actual costs and actual
activity usage levels, what frustrations might the manager feel?

Answer
The manager will not be able to plan or control costs since the charge the manager
receives will be a function of the manager’s usage and the usage of the other managers as
well as cost control in the service department.

177. Describe the difference between the traditional approach to allocating costs and activity-
based costing.

Answer
The traditional approach to allocating costs assumes that all overhead costs are
proportional to production volume. In activity-based costing, the company identifies the
activities that cause costs and allocates costs based on the volume of the activities that
caused the costs. These activities may or may not be related to production volume.

178. What is the difference between ABC and ABM?

Answer
While ABC focuses on activities with the goal of measuring the costs of products and
services produced by them, ABM focuses on activities with the goal of managing the
activities themselves.

*179. What are the four steps in ABM?

Answer
Determine major activities, identify resource used by each activity; evaluate the
performance of the activities; and identify ways to improve the efficiency and
effectiveness of the activities
CH APTE R 7
Use of Cost Information in Management Decision-Making
Summary of Questions by Objectives and Bloom’s Taxonomy
TRUE-FALSE

1. Incremental revenue is the additional revenue received as a result of selecting one


decision alternative over another.

2. Sunk costs are not incremental costs.

3. Differential costs are also incremental costs.

4. In deciding whether to sell or process further, the costs that have been incurred to process
the product to this point are incremental costs.

5. In a make or buy decision, direct materials and direct labor are usually incremental costs.

6. In a make or buy decision, the original purchase price of the equipment which is currently
used in the manufacturing process is usually a relevant cost.

7. Fixed costs that are avoidable are incremental costs in a make or buy decision.

8. Avoidable costs are not relevant.

9. Chief resource officer is another title for the person who is responsible for the company’s
financial accounting system.

10. The proper way to analyze the decision to drop a product line is to compare sunk costs to
incremental costs.

11. Common costs are fixed costs that are not directly traceable to an individual product line.

12. If a department is eliminated, the company will avoid the fixed costs that have been
allocated to that department.

13. When deciding whether to eliminate a segment, the segment should generally be kept if
its contribution margin less avoidable fixed costs is positive.

14. Opportunity costs represent the benefits foregone by selecting one alternative over
another.

15. Avoidable costs are always incremental costs.

16. Fixed costs are always sunk costs.


7-2 Test Bank to accompany Jiambalvo Managerial Accounting, 4th Edition

17. Two or more products which result from common inputs are called joint products.

18. The best way to allocate the cost of common inputs to joint products is based on the
physical quantities of the outputs.
Chapter 7 The Use of Cost Information in Management Decision Making 7-3

19. Allocating joint costs to products based on physical quantities will make the products
have the same contribution margin ratio if they are sold at the split-off point.

20. The stage of production at which individual products are identifiable is referred to as the
spin-off point.

21. A disadvantage of using an outside supplier is the associated loss of control over the
production process.

22. The qualitative aspects of a decision must receive the same careful attention as the
quantitative aspects.

23. The primary benefit of using an outside supplier is that the adverse effect of a downturn
in business is less severe.

*24. When applying the theory of constraints, management attempts to improve throughput in
all areas of the factory.

*25. Improving performance in areas which are not bottlenecks will not improve overall
output.

*26. According to the theory of constraints, everything else should be subordinate to the
binding constraint.

*27. The binding constraint is the process that limits throughput.

Material from the appendix to the chapter is marked with an asterisk (*).

Answers

1 T 6 F 11 T 16 F 21 T 26 T
2 T 7 T 12 F 17 T 22 T 27 T
3 T 8 F 13 T 18 F 23 T
4 F 9 F 14 T 19 F 24 F
5 T 10 F 15 T 20 F 25 T
7-4 Test Bank to accompany Jiambalvo Managerial Accounting, 4th Edition

MULTIPLE CHOICE

28. Which of the following is often not a differential cost?


A. Direct labor
B. Direct material
C. Variable manufacturing overhead
D. Fixed manufacturing overhead

29. Which of the following is never considered in incremental analysis?


A. Incremental revenue
B. Sunk costs
C. Incremental profit
D. Differential costs

30. Which of the following is not a term used to describe the additional costs incurred as a
result of selecting one decision alternative over another?
A. Differential costs
B. Relevant costs
C. Sunk costs
D. Incremental costs

31. When deciding between two alternatives, the preferred alternative always has
A. no opportunity costs.
B. greater revenues than the other alternatives.
C. less expense than the other alternatives.
D. greater incremental profit than the other alternatives.

32. Costs that were incurred in the past which are never incremental costs are called
A. sunk costs.
B. opportunity costs.
C. differential costs.
D. relevant costs.

33. A company is trying to decide whether to sell partially completed goods in their current
state or incur additional costs to finish the goods and sell them as complete units. Which
of the following is not relevant to the decision?
A. The selling price of the completed units.
B. The costs incurred to process the units to this point.
C. The selling price of the partially completed units.
D. The costs that will be incurred to finish the units.
Chapter 7 The Use of Cost Information in Management Decision Making 7-5

34. A company is trying to decide whether to keep or drop the sporting goods department in
its department store. If the segment is dropped, the manager will be fired. The manager's
salary, in relation to the decision to keep or drop the sporting goods department, is
A. avoidable and therefore relevant.
B. not avoidable and therefore relevant.
C. sunk and therefore not relevant.
D. the same for all alternatives and therefore not relevant.

35. Which of the following is not normally relevant in a make or buy decision?
A. direct materials
B. supervisory salaries
C. incremental revenues
D. opportunity costs

36. A company is currently making a necessary component in house (the company is


producing the component for its own use). The company has received an offer to buy the
component from an outside supplier. A machine is being rented to make the component.
If the company were to buy the component, the machine would no longer be rented. The
rent on the machine, in relation to the decision to make or buy the component, is:
A. sunk and therefore not relevant.
B. avoidable and therefore not relevant.
C. avoidable and therefore relevant.
D. unavoidable and therefore relevant.

37. Fixed costs that will be eliminated if a particular course of action is undertaken are called
A. optional costs.
B. opportunity costs.
C. direct costs.
D. avoidable costs.

38. The value of benefits foregone by selecting one decision alternative over another is a(n)
A. sunk cost.
B. incremental benefit.
C. differential revenue.
D. opportunity cost.

39. A product line should be dropped when


A. it has a negative contribution margin.
B. its avoidable fixed costs are greater than its contribution margin.
C. there will be a positive change in income if the product line is dropped.
D. All of the above.
7-6 Test Bank to accompany Jiambalvo Managerial Accounting, 4th Edition

40. Common costs


A. are fixed costs that are not directly traceable to an individual product line.
B. normally not avoidable.
C. Both A and B are true.
D. Neither A nor B is true.

41. Which of the following is a direct cost of a specific department in a retail store?
A. Company president’s salary.
B. Rent on the store.
C. Utilities for the store.
D. Cost of the department’s inventory.

42. When a department or product line is dropped, the common fixed costs which had been
allocated to that department
A. are eliminated.
B. become variable costs.
C. are allocated to the remaining departments or product lines.
D. become sunk costs.

43. You have tickets to go to Mexico (Cancun specifically) over spring break. Just this week
your best friend informs you that s/he is getting married over spring break and would like
you to be in the wedding as an attendant. Which of the following is a sunk cost that
should not be relevant to your decision as to whether be in the wedding or go on the trip
to Mexico?
A. The cost of the airline tickets to Mexico.
B. The amount of refund you could get on the airline tickets to Mexico.
C. The cost of the clothing you will have to buy/rent to be in the wedding.
D. The fact that you have never been anywhere for Spring Break and were really
looking forward to going to Mexico.

44. Which of the following statements regarding opportunity costs is true?


A. Opportunity costs are recorded with two debits, rather than a debit and a credit.
B. Opportunity costs are always incremental costs.
C. Opportunity costs are a key factor in financial accounting.
D. The same decision will be reached whether or not opportunity costs are
considered in the analysis.
Chapter 7 The Use of Cost Information in Management Decision Making 7-7

45. Robin Company currently produces 8,000 units of part B13. Current costs for part B13
are as follows:

Direct materials $12


Direct labor 9
Factory rent 7
Administrative 10
General factory overhead 7
Total $45

If the company decides to buy part B13, 50% of the administrative costs would be
avoided. All of the Robin Company items, including part B13, are manufactured in the
same rented production facility. The company has an offer from a wholesaler that wishes
to sell the part to Robin for $31 per unit. What effect will occur if the company decides to
accept the offer?
A. The cost for this part will increase by $5 per unit.
B. The cost for this part will be the same.
C. The cost for this part will decrease by $14 per unit.
D. The cost for this part will decrease by $10 per unit.

46. Walter Jewelry Company produces a bracelet which normally sells for $79.95. The
company produces 1,500 units annually but has the capacity to produce 2,000 units. A
special order for manufacturing and selling 200 bracelets at $49.95 has been received
which would not disrupt current operations. Current costs for the bracelet are as follows:

Direct materials $17.00


Direct labor 14.50
Variable overhead 4.00
Fixed overhead 5.00
Total $40.50

In addition, the customer would like to add a monogram to each bracelet which would
require an additional $2 per unit in additional labor costs and Walter Jewelry would also
have to purchase a piece of equipment to create the monogram which would cost $1,600.
This equipment would not have any other uses. With regard to this special order, only
A. incremental revenues will exceed incremental costs by $2,490.
B. incremental revenues will exceed incremental costs by $890.
C. incremental revenues will exceed incremental costs by $2,890
D. incremental revenues will exceed incremental costs by $1,290
7-8 Test Bank to accompany Jiambalvo Managerial Accounting, 4th Edition

47. NY Memorabilia Company produces a souvenir plate which normally sells for $79.95.
The company produces 1,500 plates annually but has the capacity to produce 2,000
plates. A special order for manufacturing and selling 200 plates at $49.95 has been
received which would not disrupt current operations. Current costs for the plate are as
follows:

Direct materials $17.00


Direct labor 14.50
Variable overhead 4.00
Fixed overhead 5.00
Total $40.50

In addition, the customer would like to add a date to each plate which would require an
additional $2 per plate in additional labor costs and NY Memorabilia Company would
also have to purchase a piece of equipment to create the date which would cost $1,200.
This equipment would not have any other uses. Which statement is true with regard to
this special order?
A. Incremental revenues will exceed incremental costs by $2,490.
B. Incremental revenues will exceed incremental costs by $890.
C. Incremental revenues will exceed incremental costs by $2,890.
D. Incremental revenues will exceed incremental costs by $1,290.

48. Rockwell Company owns a single restaurant which has a cantina primarily used to seat
patrons while they wait on their tables. The company is considering eliminating the
cantina and adding more dining tables. Segmented contribution income statements are as
follows and fixed costs applicable to both segments are allocated on the basis of sales.

Restaurant Cantina Total


Sales $800,000 $200,000 $1,000,000
Variable costs 475,000 160,000 635,000
Direct fixed costs 50,000 15,000 65,000
Allocated fixed costs 212,500 37,500 250,000
Net Income $ 62,500 ($12,500) $50,000

What financial effect will occur to profit if Rockwell eliminates the cantina but no more
dining customers are served?
A. Net income will increase by $12,500
B. Net income will decrease to $37,500.
C. Net income will decline by $25,000
D. Net income will be $25,000
Chapter 7 The Use of Cost Information in Management Decision Making 7-9

49. Hydra Company has two locations, downtown and at a suburban mall. During March, the
company reported total net income of $337,000 and sales of $1.2 million. The
contribution margin in the downtown store was $320,000 (40% of sales). The
contribution margin in the mall store is $200,000. Total fixed costs are $90,000 in the
downtown store and $93,000 in the mall location. How much are sales at the mall
location?
A. $400,000
B. $800,000
C. $666,667
D. Not enough information is provided to answer.

50. Collegebooks Company has two locations, downtown and on campus. During March, the
company reported net income of $164,000 and sales of $1.2 million. The contribution
margin in the downtown store was $320,000 (32% of sales). The contribution margin in
the campus store is $110,000. Direct fixed costs are $90,000 in the downtown store and
$93,000 in the campus location. How much are total variable costs?
A. $410,000
B. $3,750,000
C. $192,000
D. $853,000

51. Ricket Company has 1,500 obsolete calculators that are carried in inventory at a cost of
$13,200. If these calculators are upgraded at a cost of $9,500, they could be sold for
$22,500. Alternatively, the calculators could be sold “as is” for $9,000. What is the net
advantage or disadvantage of reworking the calculators?
A. $13,000 advantage
B. $4,000 advantage
C. $9,200 disadvantage
D. $200 disadvantage

52. BigByte Company has 12 obsolete computers that are carried in inventory at a cost of
$13,200. If these computers are upgraded at a cost of $7,500, they could be sold for
$15,300. Alternatively, the computers could be sold “as is” for $9,000. What is the net
advantage or disadvantage of reworking the computers?
A. $6,300 advantage
B. $1,200 disadvantage
C. $5,400 disadvantage
D. $3,000 advantage
7-10 Test Bank to accompany Jiambalvo Managerial Accounting, 4th Edition

53. The following are production and cost data for two products, buckets and pails.

Buckets Pails
Contribution margin per unit $450 $280
Machine set-ups needed per unit 20 14

The company can only perform 14,000 set-ups each period yet there is unlimited demand
for each product. What is the maximum contribution margin for the year?
A. $315,000
B. $35,000
C. $280,000
D. $595,000

54. The following are production and cost data for two products, A and B.

Product A Product B
Contribution margin per unit $450 $340
Machine set-ups needed per unit 25 20

The company can only perform 12,000 set-ups each period yet there is unlimited demand
for each product. What is the maximum contribution margin for the year?
A. $216,000
B. $204,000
C. $420,000
D. $18,050,000

55. Central Apparel Company owns two stores and management is considering eliminating
the east store due to declining sales. Contribution income statements are as follows and
common fixed costs are allocated on the basis of sales.
West East Total
Sales $420,000 $90,000 $510,000
Variable costs 210,000 45,000 255,000
Direct fixed costs 50,000 25,000 75,000
Allocated fixed costs 110,000 35,000 145,000
Net Income $ 50,000 ($15,000) $35,000

Central’s management feels that if they eliminate the east store, that sales in the west
store will increase by 20%. If the east store is closed, what effect will occur to the overall
company net income?
A. Increase by $25,000
B. Increase by $22,000
C. Increase by $12,000
D. Increase by $15,000
Chapter 7 The Use of Cost Information in Management Decision Making 7-11

56. Explorer Company manufactures two products, hard-tops and covers for its convertible
vehicles. Data for each follows:

Hard-top Covers
Direct labor hours required per unit 4 8
Contribution margin per unit $240 $390

Only 4,200 direct labor hours are available per month. How many units of each product
should Explorer make in order to maximize profits?
Hard-tops Covers
A. 4,200 400
B. 1,050 0
C. 1,050 250
D. 0 250

57. Urban Athletics Company has two store locations, north and south. During October, the
company reported net income of $192,000 on sales of $905,000. Sales in the north store
were $680,000 and variable costs in the south store were 60% of sales. The contribution
margin in the north store was $204,000. If total direct costs are $50,000, how much will
allocated fixed costs be?
A. $102,000
B. $2,000
C. $52,000
D. $30,000

58. The Abbott Company currently makes 10,000 units annually of a part it utilizes in the
products it manufactures. Current costs for the part are as follows:

Direct materials $16.25


Direct labor 11.85
Variable manufacturing overhead 6.30
Fixed manufacturing overhead 10.20
Total $44.60

If the company decides to buy the part the empty warehouse space could be rented for
$35,000 annually. In addition, half of the fixed manufacturing overhead costs would be
avoided if the company decides to buy the part. The company has an offer from a
manufacturer to produce the part for $42 per unit. If the company decides to accept the
offer the net advantage or disadvantage to the company’s annual net income would be:
A. An advantage of $10,000.
B. An advantage of $35,000.
C. A disadvantage of $25,000.
D. An advantage of $26,000.
7-12 Test Bank to accompany Jiambalvo Managerial Accounting, 4th Edition

59. Manor Homes plans to discontinue a segment which last year generated a contribution
margin of $65,000 and incurred $40,000 in fixed costs. If the segment is discontinued,
half of the fixed costs will not be avoided. If Manor Homes decides to discontinue this
segment the overall effect on profits will be:
A. a decrease of $65,000.
B. a decrease of $25,000.
C. a decrease of $45,000.
D. an increase of $45,000.

60. Rumper Company has 2,000 obsolete ratchers in its inventory which have a cost of $22
each. If the ratchers are reworked they could be sold for $37 each. If sold as-is, the
revenue would be only $10 each. If Rumper Company decides to rework the ratchers,
how much should the company be willing to invest to ensure that no additional loss
occurs on the sale of the ratchers?
A. $44,000
B. $54,000
C. $20,000
D. $22,000

61. Wester Company sells product Z for $23 per unit. Unit product costs are as follows:

Direct materials $4
Direct labor 5
Manufacturing overhead 12
Total $21

A special order to purchase 20,000 units was recently received. There is enough capacity
to fill the order and filling this order would not disrupt current operations. Wester
Company would incur an additional $3 per unit for shipping costs. Half of the
manufacturing overhead costs are fixed and would be incurred no matter how many units
are produced. In negotiating a price, how much is the minimum acceptable selling price?
A. $18
B. $19
C. $22
D. $15
Chapter 7 The Use of Cost Information in Management Decision Making 7-13

62. Rogertree Company manufactures a number of products from the same raw material.
Joint processing costs total $10,000 per month. Product A could be sold at the cut-off
point for $18,000 per month or it can be further processed at a cost of $9,000 per month
and then sold for $35,000. Rogertree Company should:
A. Further process product A because its incremental revenues will exceed
incremental costs by $8,000.
B. Further process product A because its incremental revenues will exceed
incremental costs by $26,000.
C. Sell as-is because the incremental loss is $2,000 if processed further.
D. Further process product A because its incremental revenues will exceed
incremental costs by $16,000.

63. Tilma Company sells product X for $23 per unit. Unit product costs are as follows:

Direct materials $4
Direct labor 5
Manufacturing overhead 12
Total $21

A special order to purchase 20,000 units was recently received. There is enough capacity
to fill the order and filling this order would not disrupt current operations. Tilma
Company would incur an additional $3 per unit for shipping costs. 40% of the
manufacturing overhead costs are fixed and would be incurred no matter how many units
are produced. In negotiating a price, how much is the minimum acceptable selling price?
A. $19.20
B. $19.00
C. $16.80
D. $12.00

64. Meadows Company manufactures a number of products from the same raw material.
Joint processing costs total $10,000 per month. Product Z could be sold at the cut-off
point for $18,000 per month or it can be further processed at a cost of $9,000 per month
and then sold for $26,000. Meadows Company should:
A. further process product Z because its incremental revenues will exceed
incremental costs by $7,000.
B. further process product Z because its incremental revenues will exceed
incremental costs by $17,000.
C. sell product Z at the split-off point because its incremental costs will exceed
incremental revenues by $1,000.
D. sell product Z at the split-off point because its incremental costs will exceed
incremental revenues by $7,000.
7-14 Test Bank to accompany Jiambalvo Managerial Accounting, 4th Edition

Answers

28 D 46 B 64 C
29 B 47 D
30 C 48 C
31 D 49 A
32 A 50 A
33 B 51 B
34 A 52 B
35 B 53 A
36 C 54 A
37 D 55 B
38 D 56 B
39 D 57 C
40 C 58 A
41 D 59 C
42 C 60 B
43 A 61 A
44 B 62 A
45 A 63 A
CHAPTER 8
Pricing Decisions
TRUE-FALSE

1. The most difficult part of determining the profit maximizing price is determining profit
at a given level of unit sales.

2. All customers are equally profitable if the same percentage markup is applied to all
customers.

3. Generally, the higher the price, the lower the quantity demanded.

4. The price which maximizes revenues is the price that should be selected.

5. The demand curves for most firms are readily known by the firm.

6. Charging a higher price per unit will always lead to higher profits for the company.

7. A company should never price below full cost.

8. Fixed costs will never affect a special order decision.

9. A company cannot increase profits if it accepts a price below full cost.

10. In order for a special order to be accepted, the full cost of the product must be covered.

11. Only incremental costs and revenues should be used when evaluating a special order.

12. A company should never accept any orders below its normal selling price.

13. It is becoming common in the airline industry to charge different customers different
prices for essentially the same flight.

14. The most difficult part of using cost-plus pricing is determining the demand function.

15. Cost plus pricing is simple to apply.

16. A company using cost-plus pricing starts with an estimate of the cost and adds a markup
to arrive at a price that allows for a reasonable level of profit.

17. Cost plus pricing is the pricing method most clearly advocated by economic theory.

18. Product cost can be largely influenced after design is complete.

19. The target costing process begins with the design of the product.
8- 2 Test Bank to accompany Jiambalvo Managerial Accounting 4th Edition

20. The first step in the target costing process is to assess the customer’s demand for the
product.
Chapter 8 Pricing Decisions 8-3

21. In target costing, the price the product will sell for is normally determined before the
cost is calculated.

22. Once a product has been designed, it is very difficult to reduce the production cost of the
product.

23. Target cost is computed by determining the appropriate selling price and subtracting
desired profit.

24. In effect, a customer profitability measurement system uses ABC to allocate costs to
customers instead of to products and services.

25. A customer profitability measurement system evaluates the profitability of customers by


taking the sales from each customer and subtracting the full manufacturing cost of
products sold to those customers.

26. A customer profitability measurement system uses activity based costing to determine
the total cost of servicing each customer.

27. A customer who makes small frequent orders is generally more profitable than a
customer paying the same price who makes larger, less frequent orders.

28. Activity based pricing will charge customers buying products for the services used to
order and delivery those products.

29. Activity based pricing will make the prices for all customers increase.

30. Suppliers adopting activity based pricing will encourage customers to demand more
services from suppliers.

31. Activity-based pricing used ABC concepts to determine the one optimum uniform price
to charge all customers.

Answers:
1 F 8 F 14 F 20 T 26 T
2 F 9 F 15 T 21 T 27 F
3 T 10 F 16 T 22 T 28 T
4 F 11 T 17 F 23 T 29 F
5 F 12 F 18 F 24 T 30 F
6 F 13 T 19 F 25 F 31 F
7 F
8- 4 Test Bank to accompany Jiambalvo Managerial Accounting 4th Edition

MULTIPLE CHOICE

32. Economic theory says to set the price that will


A. allow you to maximize profits.
B. allow you to maximize the number of units sold.
C. allow you to maximize revenues.
D. All of the above.

33. The two factors that should be considered in a product pricing decision are
A. demand and cost.
B. cost and supply.
C. competitors’ actions and demand.
D. supply and demand.

34. As the price increases, the quantity demanded will generally:


A. increase.
B. decrease.
C. stay the same.
D. increase and then decrease.

35. Which of the following stays constant when the price per unit changes?
A. Demand
B. Contribution margin per unit
C. Total fixed costs
D. Profit

36. Which of the following statements about price, demand, and profit is most generally
true?
A. As price increases, demand increases.
B. As demand increases, cost increases.
C. As price increases, demand decreases.
D. As price increases, profit decreases.

37. Which of the following statements about prices and profit is true?
A. Higher prices always lead to higher profits.
B. Higher prices always lead to lower demand and lower profits.
C. Higher prices combine with lower demand to change the level of profits.
D. Higher prices will be offset by lower demand so profits will stay constant.

38. The price for a product should be set to maximize


A. revenue.
B. contribution margin per unit.
C. profit.
D. number of units sold.
Chapter 8 Pricing Decisions 8-5

39. Billings Company sells one product with a variable cost of $4 per unit. The company is
unsure what price to charge in order to maximize profits. The price charged will also
affect the demand as shown below.

Units Sold Price


20,000 $9
30,000 $8
35,000 $7
50,000 $6

If fixed costs are $100,000 and the chart represents the demand at various prices, what
price should be charged in order to maximize profits?
A. $9
B. $8
C. $7
D. $6

40. A manufacturing company sells one product with a variable cost of $22 per unit. The
company knows that the price charged will affect demand. Fixed costs are $450,000 and
the following chart represents the estimated demand at various price levels.

Units Sold Price


45,000 $35
60,000 $33
75,000 $31
90,000 $27

If the company recorded a profit of what price was charged?


A. $35
B. $33
C. $31
D. $27

41. A manufacturing company sells one product with a variable cost of $18 per unit. The
company knows that the price charged will affect demand. Fixed costs are $275,000 and
the following chart represents the estimated demand at various price levels. If sales
exceed 50,000 units the company will need to lease additional manufacturing space and
equipment at an additional cost of $100,000 per year.

Units Sold Price


25,000 $30
50,000 $28
75,000 $26
100,000 $23

Based on this information which of the following statements is true?


8- 6 Test Bank to accompany Jiambalvo Managerial Accounting 4th Edition

A. Selling the units at $28 will generate a profit of $225,000.


B. Selling the units at $23 will generate a profit of $225,000.
C. Selling the units at $26 will generate a profit of $225,000.
D. Both A and D are correct therefore the company would record the same profit
under either pricing option.

42. The Badger Company sells one product with a variable cost of $5 per unit. The company
is unsure what price to charge in order to maximize profits. The price charged will also
affect the demand.

Units Sold Price


30,000 $10
40,000 $9
50,000 $8
60,000 $7

If fixed costs are $100,000 and the following chart represents the demand at various
prices, what price should be charged in order to maximize profits?
A. $10
B. $9
C. $8
D. $7

43. Clark Company sells a single product that has variable costs of $12 per unit and fixed
costs of $800,000. Clark estimates demand at various activity levels as follows:

Units Sold Price per Unit


100,000 $30
140,000 $28
180,000 $25
200,000 $22
240,000 $18

What price should Clark charge to maximize profits?


A. $30
B. $28
C. $25
D. $22
E. $18
Chapter 8 Pricing Decisions 8-7

44. Iguana Company sells a single product. Iguana estimates demand and costs at various
activity levels as follows:

Units Sold Price Total Variable Costs Fixed Costs


120,000 $48 $3,000,000 $1,000,000
140,000 $45 $3,500,000 $1,000,000
160,000 $40 $4,000,000 $1,000,000
180,000 $35 $4,500,000 $1,000,000
200,000 $30 $5,000,000 $1,000,000

What price should Iguana charge to maximize profits?


A. $48
B. $45
C. $40
D. $35
E. $30

45. Iguana Company sells a single product. Iguana estimates demand and costs at various
activity levels as follows:

Units Sold Price Total Variable Costs Fixed Costs


120,000 $48 $3,000,000 $1,000,000
140,000 $45 $3,500,000 $1,000,000
160,000 $40 $4,000,000 $1,000,000
180,000 $35 $4,500,000 $1,000,000
200,000 $30 $5,000,000 $1,000,000

How much profit will Iguana have if a price of $45 is charged?


A. $20
B. $6,300,000
C. $2,800,000
D. $1,800,000
E. $0

46. Iguana Company sells a single product. Iguana estimates demand and costs at various
activity levels as follows:

Units Sold Price Total Variable Costs Fixed Costs


120,000 $48 $3,000,000 $1,000,000
140,000 $45 $3,500,000 $1,000,000
160,000 $40 $4,000,000 $1,000,000
180,000 $35 $4,500,000 $1,000,000
200,000 $30 $5,000,000 $1,000,000
8- 8 Test Bank to accompany Jiambalvo Managerial Accounting 4th Edition

How would you best describe Iguana’s variable cost per unit over the range shown?
A. It is constant
B. It is increasing as volume increases
C. It is decreasing as volume increases
D. It is first increasing as volume increases, then decreasing.

47. Red Rock Company sells a single product that has variable costs of $14 per unit. Fixed
costs will remain constant across all levels of sales shown.

Units Sold Price per Unit


80,000 $35
90,000 $33
100,000 $31
110,000 $29
120,000 $27

What price should Red Rock charge to maximize profits?


A. $35
B. $33
C. $31
D. $29
E. $27

48. Cherokee Company sells a single product that has variable costs of $10 per unit. Fixed
costs will be $700,000 across all levels of sales shown.

Units Sold Price per Unit


80,000 $35
90,000 $33
100,000 $31
110,000 $30
120,000 $28

What price should Cherokee charge to maximize profits?


A. $35
B. $33
C. $31
D. $30
E. $28
Chapter 8 Pricing Decisions 8-9

49. Cherokee Company sells a single product that has variable costs of $10 per unit. Fixed
costs will be $700,000 across all levels of sales shown.

Units Sold Price per Unit


80,000 $35
90,000 $33
100,000 $31
110,000 $30
120,000 $28

What, if any information given was not relevant to the price maximization decision?
A. The selling prices.
B. The variable costs per unit.
C. The quantities demanded.
D. The total fixed costs.
E. All of the information was relevant.

50. Cherokee Company sells a single product that has variable costs of $10 per unit. Fixed
costs will be $700,000 across all levels of sales shown.

Units Sold Price per Unit


80,000 $35
90,000 $33
100,000 $31
110,000 $30
120,000 $28

What price would Cherokee charge to maximize revenues?


A. $35
B. $33
C. $31
D. $30
E. $28

51. Jackson Company is trying to determine the optimal price to charge for its PUNCH
model. Jackson has fixed costs of $50,000 and the PUNCH has variable costs of $12.00
per unit. Jackson has determined that the following relationships exist between price and
demand:

Price Demand
$20 6,875
$19 8,800
$18 10,000
$17 11,000
8- 10 Test Bank to accompany Jiambalvo Managerial Accounting 4th Edition

What is the contribution margin for a price of $20?


A. $12.00
B. $10.00
C. $8.00
D. $6.00

52. Jackson Company is trying to determine the optimal price to charge for its PUNCH
model. Jackson has fixed costs of $50,000 and the PUNCH has variable costs of $12.00
per unit. Jackson has determined that the following relationships exist between price and
demand:

Price Demand
$20 6,875
$19 8,800
$18 10,000
$17 11,000

What is the anticipated revenue for a price of $19?


A. $167,200
B. $137,500
C. $350,000
D. $155,000

53. Jackson Company is trying to determine the optimal price to charge for its PUNCH
model. Jackson has fixed costs of $50,000 and the PUNCH has variable costs of $12.00
per unit. Jackson has determined that the following relationships exist between price and
demand:

Price Demand
$20 6,875
$19 8,800
$18 10,000
$17 11,000

What is the anticipated profit for a price of $18?


A. $5,000
B. $10,000
C. $12,000
D. $60,000
Chapter 8 Pricing Decisions 8-11

54. Jackson Company is trying to determine the optimal price to charge for its PUNCH
model. Jackson has fixed costs of $50,000 and the PUNCH has variable costs of $12.00
per unit. Jackson has determined that the following relationships exist between price and
demand:

Price Demand
$20 6,875
$19 8,800
$18 10,000
$17 11,000

What is the anticipated profit for a price of $17?


A. $55,000
B. $10,000
C. $12,000
D. $5,000

55. Jackson Company is trying to determine the optimal price to charge for its PUNCH
model. Jackson has fixed costs of $50,000 and the PUNCH has variable costs of $12.00
per unit. Jackson has determined that the following relationships exist between price and
demand:

Price Demand
$20 6,875
$19 8,800
$18 10,000
$17 11,000

What price should Jackson charge in order to maximize its profit?


A. $20
B. $19
C. $18
D. $17

56. Betty Boulder is the owner/operator of a tanning salon. She is considering four price
levels for a weekly tanning pass. Her estimate of price and quantity demanded are:

Price Quantity Demanded


$14.00 310
$12.00 380
$10.00 420
$7.00 530
8- 12 Test Bank to accompany Jiambalvo Managerial Accounting 4th Edition

Monthly costs of providing the tanning service include $2,500 of fixed costs and
variable costs of $2.50 per service. Which price will yield the largest monthly profit?
A. $14.00
B. S12.00
C. $10.00
D. $7.00

57. Cortez Corporation has analyzed its customer and order handling data for the past year
and has determined the following costs:

Order processing cost per order $6


Additional costs if order must be expedited (Rushed) $9
Customer technical support calls (Per call) $7
Relationship management costs (Per customer per year) $1,900

In addition to these costs, product costs amount to 80 percent of sales. In the prior year,
Cortez had the following experience with one of its customers, Green Company:

Sales $25,000
Number of orders 170
Percent of orders marked rush 80
Calls to technical support 90

Calculate the profitability of the Green Company account.


A. $20,226
B. $226
C. $1,260
D. Some other answer

58. Eager Electronics has just developed a low-end electronic calendar that it plans on
selling via a cable channel marketing program. The cable program’s fee for selling the
item is 20 percent of revenue. For this fee, the program will sell the calendar over six
10-minute segments in September.
Eager’s fixed costs of producing the calendar are $110,000 per production run. The
company plans to wait for all orders to come in, and then it will produce exactly the
number of units ordered. Variable production costs are $25 per unit. In addition, it will
cost approximately $5 per unit to ship the calendars to customers. Production time will
be less than three weeks. Bill Colt, a product manager at Eager, is charged with
recommending a price for the item. Based on his experience with similar items, focus
group responses, and survey information, he has estimated the number of units that can
be sold at various prices:
Chapter 8 Pricing Decisions 8-13

Price Quantity
$74.99 15,000
$69.99 20,000
$54.99 25,000
$49.99 35,000

At what price is profit maximized?


A. $74.99
B. $69.99
C. $54.99
D. $49.99

59. Eager Electronics has just developed a low-end electronic calendar that it plans on
selling via a cable channel marketing program. The cable program’s fee for selling the
item is 12 percent of revenue. For this fee, the program will sell the calendar over six
10-minute segments in September.
Eager’s fixed costs of producing the calendar are $62,000 per production run. The
company plans to wait for all orders to come in, and then it will produce exactly the
number of units ordered. Production time will be less than three weeks. Variable
production costs are $14 per unit. In addition, it will cost approximately $2 per unit to
ship the calendars to customers. Wonyoni Fletcher, a product manager at Eager, is
charged with recommending a price for the item. Based on her experience with similar
items, focus group responses, and survey information, she has estimated the number of
units that can be sold at various prices:

Price Quantity
$42 5,000
$38 8,000
$34 11,000
$30 13,000

At which price should the company sell it products?


A. $42
B. $38
C. $34
D. $30

60. Citrus Company sells its single product for $60 per unit. Unit product costs are as
follows:

Direct materials $20


Direct labor 20
Manufacturing overhead 4
Total $44
8- 14 Test Bank to accompany Jiambalvo Managerial Accounting 4th Edition

A special order to purchase 10,000 units was recently received. There is enough capacity
to fill the order. Filling this order would not disrupt current operations. The Citrus
Company would incur an additional $5 per unit for additional labor costs due to a slight
modification the buyer wants made to the original product. One-fourth of the
manufacturing overhead costs are fixed and would be incurred no matter how many
units are produced. In negotiating a price, how much is the minimum acceptable selling
price?
A. $44
B. $45
C. $48
D. $60

61. Which of the following should be true in order for a company to accept a special order?
A. Variable costs are less than fixed costs.
B. Incremental revenues are greater than incremental costs.
C. Opportunity costs are zero.
D. The order is for a current customer.

62. When deciding to accept or reject a special order, which of the following costs would
most likely not be relevant?
A. The wages of direct labor to make the order.
B. Depreciation on the machinery used to make the order.
C. The raw material used to make the order.
D. The electricity used to run the machine to make the order.

63. In accepting a special order, the company should consider whether


A. production capacity exists to complete the order.
B. demand for other products will not be adversely affected.
C. the incremental costs are less than the incremental revenues.
D. All of the above are true.

64. Which of the following are relevant in deciding whether to accept or reject a special
order?
A. The impact the order will have on existing business.
B. The price that will be charged on the special order.
C. The incremental cost of filling the special order.
D. All of the above.
Chapter 8 Pricing Decisions 8-15

Answers

32 A 52 A
33 D 53 B
34 B 54 D
35 C 55 B
36 C 56 B
37 C 57 B
38 C 58 B
39 B 59 C
40 C 60 C
41 D 61 B
42 B 62 B
43 C 63 D
44 B 64 D
45 D
46 A
47 B
48 D
49 D
50 E
51 C
CH APTE R 10
Budgetary Planning and Control
True-False

1. A budget is a formal document that quantifies a company’s plans for achieving its goals.

2. Budgets are useful in the control process because they provide a basis for evaluating
performance.

3. A top-down approach to budgeting involves substantial input from lower level managers.

4. Most managers believe that budgeting is more successful when a top-down approach
rather than a bottom-up approach is used.

5. Generally, the longer the time period involved, the more detailed the budget.

6. An advantage of zero base budgeting is that it results in a fresh consideration of the


validity of budget amounts.

7. Only manufacturing firms need to prepare a budget.

8. Any significant deviation from planned performance is associated with managers doing a
good or poor job managing operations.

9. The first step in the budget process is preparing the sales forecasts.

10. If the number of units produced equals the number of units sold, the number of units in
ending inventory will equal the number of units in beginning inventory.

11. The sales budget is constructed after the production budget is finalized.

12. All of the dollar amounts in the manufacturing overhead budget represent cash flows in
the period incurred.

13. Items in the capital acquisitions budget appear in the cost of goods sold section of the
budgeted income statement.

14. The amount and timing of cash flows is the focus of the cash receipts and disbursements
budget.

15. A company will often have cash flow problems ahead of a period of increasing sales.

16. Just in time inventory eliminates the need for budgeting.

17. The budgeted balance sheet is also called a pro-forma balance sheet.
10-2 Test Bank to accompany Jiambalvo Managerial Accounting, 4th Edition

18. Using spreadsheet based computer programs will help a company perform “what if”
budget analysis.
Chapter 10 Budgetary Planning and Control 10-3

19. The selling and administrative expense budget is based on the numbers in the production
budget.

20. Differences between budgeted and actual amounts are referred to as flexible budgets.

21. A static budget is prepared for a single anticipated level of production.

22. If volume differs from the original static budget, it is unfair to evaluate cost performance
against that budget.

23. A flexible budget is a set of budget relationships that can be adjusted to various activity
levels.

24. In a management by exception approach, only unfavorable variances will be investigated.

25. Managers may be tempted to move revenues and expenses between time periods to meet
performance targets.

26. Generally, it is best to evaluate managers against a single budget number.

27. Waiting until January 1 to ship an order that was ready on December 29 is an example of
income shifting.

28. When the same budget is used for planning and control, managers may be tempted to
build slack into their budgets.

29. There is an inherent conflict when budgets are used for both planning and control.

30. Only items that can be measured in monetary terms should be budgeted and used for
control purposes.

Answers
1 T 7 F 13 F 19 F 25 T
2 T 8 F 14 T 20 F 26 F
3 F 9 T 15 T 21 T 27 T
4 F 10 T 16 F 22 T 28 T
5 F 11 F 17 T 23 T 29 T
6 T 12 F 18 T 24 F 30 F
10-4 Test Bank to accompany Jiambalvo Managerial Accounting, 4th Edition

MULTIPLE CHOICE

31. A budget
A. is the formal document that quantifies a company’s plans for achieving its goals.
B. enhances communication within a company.
C. provides a basis for evaluating performance.
D. All of the above are true.

32. The formal documents that quantify a company’s plans for achieving its goals are called
A. variance reports.
B. budgets.
C. exception logs.
D. cost of production reports.

33. A budget is useful in the planning process because it


A. evaluates performance
B. forces managers to think about goals and objectives and means of achieving them
C. both A and B
D. neither A nor B

34. Development of a budget


A. is required by GAAP.
B. is a task best completed by the controller working alone.
C. enhances communication and coordination among managers.
D. guarantees that the company will be profitable.

35. Which of the following is not a reason that actual results may deviate from planned
performance?
A. A bottom-up approach to budgeting was used.
B. Managers have done a particularly good or particularly poor job of managing
operations.
C. Conditions have changed since the budget was developed.
D. The budget was poorly conceived and constructed.

36. The person evaluating a manager should consider


A. any deviation from budgeted amounts as an item that should be investigated.
B. all favorable variances as indications of good performance.
C. that managers will focus their attention on those measures that they know will be
part of their evaluation.
D. All of the above are true.

37. The group that is responsible for development and approval of the budget is the:
A. budget committee.
B. the union.
C. the auditors.
D. the board of directors.
Chapter 10 Budgetary Planning and Control 10-5

38. The budget committee consists of


A. senior managers, including the CEO and CFO.
B. representatives from the stockholders and suppliers.
C. all those who have loaned money to the company.
D. all employees interested in providing input to the budgeting process.

39. In a bottom-up approach to budgeting


A. the CFO alone determines the budget.
B. lower level managers are the primary source of information used in setting the
budget.
C. the production budget is developed before the sales budget.
D. only the budget for the next month can be prepared.

40. In a bottom-up approach to budgeting, the primary source of information used in setting
the budget is
A. economists.
B. trade magazines.
C. the controller.
D. lower level managers.

41. Which of the following statements regarding approaches to budgeting is true?


A. Most managers believe that successful budgeting requires a bottom-up approach.
B. A top-down approach does not involve substantial input from lower level
managers.
C. Both A and B are true.
D. Neither A nor B is true.

42. Less detailed budgets are associated with


A. production costs.
B. non-profit organizations.
C. longer time periods.
D. zero base budgeting.

43. A method of budget preparation that requires all budgeted amounts to be justified by the
department, even if the amounts were supported in prior periods, is called
A. variance budgeting.
B. flexible budgeting.
C. current period budgeting.
D. zero base budgeting.

44. Which of the following is not a characteristic of zero base budgeting?


A. It is inconsistent with bottom-up budgeting.
B. It is expensive.
C. It has gained considerable support in governmental budgeting.
D. It results in a fresh consideration of the validity of budget amounts.
10-6 Test Bank to accompany Jiambalvo Managerial Accounting, 4th Edition

45. The comprehensive planning document that incorporates a number of individual budgets
is the
A. capital acquisitions budget.
B. master budget.
C. material purchases budget.
D. sales forecast.

46. Which of the following is not typically part of the master budget?
A. Direct material purchases budget.
B. Variance reports.
C. Projected cash receipts and disbursements.
D. A pro-forma balance sheet.

47. The master budget incorporates individual budgets such as those for
A. direct materials, direct labor, and selling and administrative expenses.
B. best case, worst case, and most likely sales forecasts.
C. one year ago, five years ago, and ten years ago.
D. each company in the industry.

48. Which of the following is a reasonable order in which to prepare budgets?


A. Budgeted income statement, sales budget, cash receipts and disbursements
budget.
B. Cash receipts and disbursements budget, capital acquisitions budget, labor budget.
C. Sales budget, production budget, material purchases budget.
D. Labor budget, budgeted income statement, sales budget.

49. Which of the following budgets is prepared last?


A. Sales budget.
B. Capital acquisitions budget.
C. Material purchases budget.
D. Budgeted balance sheet.

50. Which of the following budgets is prepared first?


A. Cash receipts and disbursements budget.
B. Sales budget.
C. Selling and administrative expense budget.
D. Budgeted balance sheet.

51. Companies estimate sales by using


A. models developed by economists.
B. trends in their own sales data.
C. estimates from their sales force.
D. All of the above.
Chapter 10 Budgetary Planning and Control 10-7

52. Which of the following is not a method that can reasonably be used to forecast sales?
A. Trends in the company’s sales data.
B. Flexible budgets for various levels of production.
C. Estimates from the company’s salespersons.
D. Mathematical models adjusted by an experienced manager using professional
judgment.

53. The sales budget is based on assumptions about the


A. number of units to be sold and selling price per unit.
B. timing of cash receipts.
C. contribution margin per unit and the number of units to be sold.
D. costs of the units produced and the total fixed costs.

54. Setting the sales budget is very important because


A. the rest of the master budget is driven by the sales budget.
B. many performance targets are set by the sales budgets.
C. bonuses are often at stake based on achieving the sales budget.
D. All of the above.

55. Which of the following is not used in deciding how many units to produce in a period?
A. The desired number of units in ending inventory.
B. The expected sales in units.
C. The number of units in beginning inventory.
D. The number of units of raw material in inventory.

56. Concerning relationship between beginning inventory, ending inventory, production, and
sales, which of the following is true?
A. Production = Beginning Inventory + Sales – Ending Inventory
B. Production = Sales + Ending Inventory – Beginning Inventory
C. Production = Beginning Inventory + Ending Inventory – Sales
D. Production = Beginning Inventory – Ending Inventory + Sales

57. If the number of units in beginning inventory is more than the number of units in ending
inventory, the number of units sold is
A. less than the number of units produced.
B. greater than the number of units produced.
C. less than the number of units in beginning inventory.
D. greater than the number of units in ending inventory.

58. When constructing the production budget, the desired ending inventory for the first
period is
A. the beginning inventory for the second period.
B. often expressed as a percentage of the first period’s sales.
C. generally more than the beginning inventory for the first period.
D. not a factor in the production budget.
10-8 Test Bank to accompany Jiambalvo Managerial Accounting, 4th Edition

59. The amount of direct material that must be purchased during a period depends on the
amount of direct material
A. needed for production.
B. available in the beginning inventory.
C. desired as ending inventory.
D. All of the above are correct.

60. A significant difference between the direct material purchases budget and the direct labor
budget is that the direct material purchases budget
A. is based on units sold, while the direct labor budget is based on units produced.
B. considers beginning and ending inventory amounts, which are not part of a direct
labor budget.
C. is constructed for each quarter, while the direct labor budget is constructed for
each pay period.
D. is constructed from the top down, while the direct labor budget uses a bottom-up
approach.

61. Which of the following is likely to increase the amount budgeted for depreciation in the
manufacturing overhead budget?
A. Sale of the building housing the company headquarters.
B. Increased variable costs.
C. Planned acquisitions of new equipment.
D. A decrease in the number of units produced.

62. What is one reason the amount of cash paid out for overhead each period does not equal
the total overhead incurred?
A. Depreciation is an overhead expense that does not require the use of cash.
B. Overhead expenses are only estimates, they do not require cash.
C. Cash is only paid out for fixed manufacturing overhead expenses.
D. The amount of cash paid out is adjusted for the number of units sold.

63. Which of the following does not appear on the budgeted income statement?
A. Cost of goods sold.
B. Sales.
C. Selling and administrative expenses.
D. Accounts receivable.

64. Which of the following is not required in order to calculate the cost of goods sold in the
budgeted income statement?
A. Number of units sold.
B. Direct material and direct labor costs per unit.
C. Manufacturing overhead cost per unit.
D. Selling and administrative costs per unit.
Chapter 10 Budgetary Planning and Control 10-9

Answers
31 D 53 A
32 B 54 D
33 C 55 D
34 C 56 B
35 A 57 B
36 C 58 A
37 A 59 D
38 A 60 B
39 B 61 C
40 D 62 A
41 C 63 D
42 C 64 D
43 D
44 A
45 B
46 B
47 A
48 C
49 D
50 B
51 D
52 B

You might also like